www.insightsonindia.com www.insightsias.com TEST – 5 Solutions

Prelims 2016 – Test 5 SOLUTIONS

1. Consider the following about the World Sustainable Development Summit (WSDS), 2016 that was formerly organized as the Delhi Sustainable Development Summit (DSDS). 1. This is the first such edition of the WSDS. 2. It is organized by the Ministry of New and Renewable Energy (MNRE), Government of . 3. The 2016 event is themed as “Beyond 2015: People, Planet & Progress”. 4. It is chaired by the Prime Minister of India.

Select the correct answer using the codes below.

a) 1, 2 and 3 only b) 2 and 4 only c) 1 and 3 only d) 1, 2, 3 and 4 Solution: c)

Justification: Statement 1 and 2: It is the annual event of the Energy and Resources Institute’s (TERI). Formerly known as the Delhi Sustainable Development Summit (DSDS), it has evolved to the World Sustainable Development Summit (WSDS) in 2016 for the first time.

The Delhi Sustainable Development Summit was initiated in 2001. WSDS 2016 will be held in India Habitat Centre, New Delhi from 5th-8th October, 2016.

The WSDS will bring together thinkers (including Nobel laureates) and leaders (heads of states) and members of civil society; to deliberate on issues related to sustainable development.

Statement 3: 2016, themed, 'Beyond 2015: People, Planet & Progress', will be among the first international platforms to discuss the new agenda

INSIGHTS PRELIMS TEST SERIES 2017 www.insightsonindia.com www.insightsias.com TEST – 5 Solutions post the adoption of the Sustainable Development Goals and the signing of the Paris Agreement.

The attempt will be to initiate discussions on methodologies to be adopted to chalk a path to realise the Goals beyond 2015.

Q Source: Current affairs: http://wsds.teriin.org/index.php

2. Consider the following about The Large Hadron Collider (LHC). 1. The term hadron in LHC refers to composite particles composed of quarks held together by the gravitational force. 2. To allow for the collision of high energy particles, cryogenic stage temperature is maintained inside the accelerator. 3. It consists of a long ring of superconducting magnets with a number of accelerating structures to boost the energy of the particles along the way.

Select the correct answer using the codes below.

a) 1 and 2 only b) 2 and 3 only c) 1 and 3 only d) 3 only Solution: b)

Justification: Statement 1: These quarks cannot be held together by such a weak gravitational force. It is the “Strong Force” (electromagnetic) which holds them together.

Statement 2 and 3: Inside the accelerator, two high-energy particle beams travel at close to the speed of light before they are made to collide. The beams travel in opposite directions in separate beam pipes – two tubes kept at ultrahigh vacuum.

They are guided around the accelerator ring by a strong magnetic field maintained by superconducting electromagnets.

INSIGHTS PRELIMS TEST SERIES 2017 www.insightsonindia.com www.insightsias.com TEST – 5 Solutions

The electromagnets are built from coils of special electric cable that operates in a superconducting state, efficiently conducting electricity without resistance or loss of energy. This requires chilling the magnets to 271.3°C – a temperature colder than outer space!

Q Source: Q100: SET A: CAPF 2014

3. How is the Admiralty (Jurisdiction and Settlement of Maritime Claims) Bill, 2016 significant in the form approved by the Cabinet? 1. It will allow India to extract minerals in competing Exclusive Economic Zones (EEZs) without the consent of the International Seabed Authority (ISA). 2. It will extend the jurisdiction of High Courts located in coastal states of India upto territorial waters.

Which of the above is/are correct?

a) 1 only b) 2 only c) Both 1 and 2 d) None Solution: b)

Justification: India is a leading maritime nation and maritime transportation caters to about 95% of its merchandise trade volume.

• However, the admiralty jurisdiction of Indian courts under the present statutory framework flow from laws enacted in the British era. • Admiralty jurisdiction is related to the powers of the High Courts in respect of claims associated with transport by sea and navigable waterways. • The repealing of five archaic admiralty statutes is in line with the Union Government’s commitment to do away with archaic laws which are hindering efficient governance.

INSIGHTS PRELIMS TEST SERIES 2017 www.insightsonindia.com www.insightsias.com TEST – 5 Solutions

• The Bill consolidates the existing laws relating to admiralty proceedings on maritime claims, admiralty jurisdiction of courts, arrest of vessels and related issues.

Q Source: http://pib.nic.in/newsite/PrintRelease.aspx?relid=150973

4. The Chinar tree, which grows throughout the Kashmir Valley, attracts a lot of attention of visitors and locals during a) Autumn Season b) Winter Season c) Monsoon Season d) Spring Season Solution: a)

Learning: It is the state tree of J&K. It traces its origin to Greece.

Chinar tree is green during the summer. But it doesn’t attract crowds as much as it draws during autumn. In this seasons its leaves acquire varied colourful hues — of blood-red, mauve, amber and yellow — in a short period of time, somewhere around October end, and remain that way till the end of November.

The majestic Chinar trees grow as tall as 25 meters, with girth exceeding 50 feet in some cases. These trees figure notably in Kashmir’s literature and politics, religion and romance.

INSIGHTS PRELIMS TEST SERIES 2017 www.insightsonindia.com www.insightsias.com TEST – 5 Solutions

Q Source: Syllabus: State birds, animals, trees and flowers

5. Lack of “Internal democracy” within political parties implies 1. Concentration of power at the top in the party 2. Provincial decentralization of the party

Which of the above is/are correct?

a) 1 only b) 2 only c) Both 1 and 2 d) None Solution: a)

• Justification: All over the world there is a tendency in political parties towards the concentration of power in one or few leaders at the top. Parties do not keep membership registers, do not hold

INSIGHTS PRELIMS TEST SERIES 2017 www.insightsonindia.com www.insightsias.com TEST – 5 Solutions

organisational meetings, and do not conduct internal elections regularly. • Ordinary members of the party do not get sufficient information on what happens inside the party. • They do not have the means or the connections needed to influence the decisions. As a result the leaders assume greater power to make decisions in the name of the party. • Since one or few leaders exercise paramount power in the party, those who disagree with the leadership find it difficult to continue in the party. More than loyalty to party principles and policies, personal loyalty to the leader becomes more important.

Q Source: Page 83: Democratic Politics: Class Xth NCERT

6. A very interesting feature of medieval India was the sarais which ringed cities and dotted a vast space of the Indian subcontinent. Sarais were a) Voluntary bodies that took care of a city’s law and order b) Local private godowns which provided grain storage facility to adjoining farms c) Places that provided temporary accommodation to travellers and pilgrims d) Dargahs that hosted religious and spiritual discussions Solution: c)

Learning: Sarais were largely built on a simple square or rectangular plan and were meant to provide temporary accommodation to Indian and foreign travellers, pilgrims, merchants, traders, etc.

In effect, sarais were public domains which thronged with people of varied cultural backgrounds. This led to cross-cultural interaction, influences and syncretic tendencies in the cultural mores of the times and at the level of the people.

INSIGHTS PRELIMS TEST SERIES 2017 www.insightsonindia.com www.insightsias.com TEST – 5 Solutions

Q Source: Improvisation: Page 115: Introduction to Indian Art – Class XI

7. Consider the following about the Office of the UPSC Chairman. 1. She is appointed by the Department of Personnel and Training (DoPT) on the recommendation of UPSC members. 2. The Chairman must serve a fixed term post-appointment even if retirement is due. 3. She can be removed by a resolution passed by a majority of UPSC members and agreed to by the Union Cabinet. 4. A UPSC member can be re-appointed as the Chairman.

Select the correct answer using the codes below.

a) 1 and 2 only b) 4 only c) 2, 3 and 4 only d) None of the above Solution: b)

Justification: Statement 1: She is appointed by the President on the recommendation of the Union Council of Ministers.

Statement 2: Smt. Alka Sirohi will replace Deepak Gupta who demits office on 20th September 2016. She will be in office till completion of her term as member on January 3, 2017. This shows that there is no fixed tenure that a Chairman must serve post-appointment. The chairman and members of the Commission hold office for a term if six years or until they attain the age of 65 years, whichever is earlier.

Statement 2: He can be removed by the President based on a Supreme Court inquiry.

Statement 4: Smt. Alka Sirohi, former UPSC member was recently appointed as UPSC Chairman.

INSIGHTS PRELIMS TEST SERIES 2017 www.insightsonindia.com www.insightsias.com TEST – 5 Solutions

Q Source: Appointment of new UPSC Chairman

8. India has officially declared itself free from which of the following diseases? 1. Chikungunya 2. Polio 3. Yaws 4. Avian Influenza 5. Leprosy 6. Smallpox

Select the correct answer using the codes below.

a) 2, 3 and 4 only b) 1, 2 and 5 only c) 2, 3, 4, 5 and 6 only d) 1, 3, 4 and 6 only Solution: c)

Justification: Statement 1: Many cases and deaths were recently reported in Delhi. India is not free from the disease which is carried mainly by mosquitoes.

Statement 2 and 3: India has become the first country in the world to get the Yaws-free certificate by the WHO.

This is the third major momentous public health feat for India, which was declared polio-free two years ago, elimination of maternal and neonatal tetanus early this year and now becoming free from Yaws, a chronic bacterial infection caused by Treponema pallidum subsp Pertenue.

Statement 4: India has officially declared itself free from highly contagious Avian Influenza (H5N1) or bird flu and has notified the same to the World Organization for Animal Health (OIE).

INSIGHTS PRELIMS TEST SERIES 2017 www.insightsonindia.com www.insightsias.com TEST – 5 Solutions

Statement 5: Leprosy was officially declared to be eradicated in 2005, but many cases are still found in India.

Statement 6: Smallpox was declared by WHO to be eradicated in 1979 from the World.

Q Source: Improvisation: Current Affairs: H5N1 eradication declaration by India

9. The “13th Amendment to Sri Lankan Constitution” is sometimes seen in news. Which major issue it is related to? a) Sharing of heritage island territory of Sri Lanka with India b) Power sharing arrangement with the Indian government in Palk Bay c) Political rights of Tamils in Sri Lanka d) Sri Lankan Military cooperation with South Asian neighbours Solution: c)

Learning: In 1987, Indo-Sri Lanka Accord was signed between Indian Prime Minister Rajiv Gandhi and Sri Lankan President J.R. Jayewardene which stated the devolution of powers to the SL provinces (Tamil).

• The amendment aims at creating provincial councils in Sri lanka and enable Sinhalese and Tamil as national languages while preserving English as the link language.

• However there are practical problems in devolving land, police and financial powers to the provinces and the Government has stressed that the structure that is implemented should be acceptable to all parts of the country.

• In February 2016, the Chief Minister of Sri Lanka's Northern Provice, C.V. Wigneswaran sought India's direct intervention in implementation of the amendment.

INSIGHTS PRELIMS TEST SERIES 2017 www.insightsonindia.com www.insightsias.com TEST – 5 Solutions

Q Source: Improvisation: Page 3: Democratic Politics: Class Xth NCERT

10. Consider the following about the Vienna Convention for the Protection of the Ozone Layer. 1. It entered in force after Montreal protocol. 2. It includes legally binding reduction goals for the use of CFCs. 3. It has been ratified by all United Nations Member states.

Select the correct answer using the codes below.

a) 1 and 2 only b) 2 and 3 only c) 3 only d) 1, 2 and 3 Solution: b)

Justification: Statement 2: It acts as a framework for the international efforts to protect the ozone layer. These are laid out in the accompanying Montreal Protocol. However, it does not include legally binding reduction goals for the use of CFCs, the main chemical agents causing ozone depletion.

Statement 1 and 3: It was agreed upon at the 1985 Vienna Conference and entered into force in 1988 (Montreal in 1989). It is one of the most successful treaties of all time in terms of universality. It has been ratified by 197 states (all UN members as well as the Niue, Holy See and the Cook Islands) as well as European Union.

Learning: Montreal Protocol on Substances that Deplete targets phasing out production of numerous Ozone Depleting Substances (ODSs).

INSIGHTS PRELIMS TEST SERIES 2017 www.insightsonindia.com www.insightsias.com TEST – 5 Solutions

Under it production and consumption of key ODSs like chlorofluorocarbon (CFCs), Methyl Chloroform, CTC halons and Methyl Bromide have been phased-out globally.

Q Source: 16th September: International Day for the Preservation of the Ozone Layer

11. The following were in news for some time. Consider their matches with what they are associated with. 1. Tiangong-2 : World’s fastest Supercomputer 2. Cassini–Huygens : NASA’s Spacecraft to study Saturn 3. NASA’s New Horizons : Interplanetary space probe 4. NASA’s Odyssey : Exploration of Jupiter

Select the correct answer using the codes below.

a) 1 and 2 only b) 2, 3 and 4 only c) 2 and 3 only d) 1 and 4 only Solution: c)

Justification: Statement 1: China has successfully launched Tiangong- 2 space lab to develop expertise for a future space station and conduct science experiments.

Statement 2: NASA’s Cassini spacecraft after studying Saturn, its rings and moons for more than 12 years, has entered in the final year of its epic voyage. It will make the closest-ever observations of the planet. It is the first to enter successfully in its orbit.

Statement 3: New Horizons is an interplanetary space probe that was launched as a part of NASA’s New Frontiers program. It was launched on January 19, 2006.

INSIGHTS PRELIMS TEST SERIES 2017 www.insightsonindia.com www.insightsias.com TEST – 5 Solutions

The primary mission is to perform a flyby study of the Pluto system. The secondary mission to fly by and study one or more other Kuiper belt objects (KBOs).

Statement 4: It is a Mars exploration spacecraft.

Q Source: Improvisation: Current Affairs: China launches Tiangong-2 space lab

12. These are water channels that are found in precipitous mountain areas of Himachal Pradesh for carrying water from glaciers to villages in the Spiti valley. The statement refers to? a) Zabo b) Khatri c) Naula d) Kulhs Solution: d)

Learning: Parts of Himachal Pradesh had evolved a local system of canal irrigation called kulhs over four hundred years ago. The water flowing in the streams was diverted into man-made channels which took this water to numerous villages down the hillside.

The management of the water flowing in these kulhs was by common agreement among all the villages. Interestingly, during the planting season, water was first used by the village farthest away from the source of the kulh, then by villages progressively higher up. These kulhs were managed by two or three people who were paid by the villagers.

In addition to irrigation, water from these kulhs also percolated into the soil and fed prings at various points. After the kulhs were taken over by the Irrigation Department, most of them became defunct and there is no amicable sharing of water as before.

INSIGHTS PRELIMS TEST SERIES 2017 www.insightsonindia.com www.insightsias.com TEST – 5 Solutions

Q Source: Page 274: Science Xth Standard NCERT

13. It is the largest protected area in the Eastern Himalaya biodiversity hotspot and harbours the northernmost lowland evergreen rainforests in the world. The habitat changes with increasing altitude from tropical moist forests to Montane forests, temperate forests and at the higher elevations to Alpine meadows and perennial snow. The national park is? a) Namdapha National Park b) Dachigam National park c) Nokrek National park d) Khangchendzonga National Park Solution: a)

Learning: The park is located in Changlang district of the Northeastern state of Arunachal Pradesh, near its border with Myanmar.

The park has extensive bamboo forests and secondary forests in addition to the primary forests.

Because of many different vegetation zones, the park is home to a great diversity of mammal species. snow leopards, clouded leopards, large indian civet, barking deer etc.

Q Source: Syllabus: http://www.worldwildlife.org/places/eastern- himalayas

14. Which of the following statements about Temple Architecture in India is INCORRECT?

INSIGHTS PRELIMS TEST SERIES 2017 www.insightsonindia.com www.insightsias.com TEST – 5 Solutions

a) Nagara and Dravida are famous South Indian temple construction styles. b) Vesara order was created through the selective mixing of Nagara and Dravida order. c) Panchayatana style of architecture was practiced during Gupta period. d) All of the above Solution: a)

Justification: Nagara is North Indian style; Dravida is South Indian. Vesara is a mixture of both.

Option C: Temple at Deograh (Lalitpur, UP) is in the panchayatana style of architecture where the main shrine is built on a rectangular plinth with four smaller subsidiary shrines at the four corners (making it a total number of five shrines, hence the name, panchayatana).

Q Source: Chapter 6: Introduction to Indian Art – Class XI

15. Having two eyes, instead of only one, accords human beings which of the following vision-related advantages? 1. Depth of field can be perceived more correctly with two eyes. 2. Width of fields increases with two eyes.

Which of the above is/are correct?

a) 1 only b) 2 only c) Both 1 and 2 d) None Solution: c)

Justification: There are several advantages of our having two eyes Instead of one.

• It gives a wider field of view. A human being has a horizontal field of view of about 150 degrees with one eye and of about 180 degrees

INSIGHTS PRELIMS TEST SERIES 2017 www.insightsonindia.com www.insightsias.com TEST – 5 Solutions

with two eyes. The ability to detect faint objects is thus enhanced with two detectors instead of one.

• Some animals, usually prey animals, have their two eyes positioned on opposite sides of their heads to give the widest possible field of view.

• Shut one eye and the world looks flat two-dimensional. Keep both eyes open and the world takes on the third dimension of depth. Because our eyes are separated by a few centimetres each eye sees a slightly different image.

• Our brain combines the two images into one, using the extra information to tell us how close or far away things are.

Q Source: Chapter 11: Science Xth Standard NCERT

16. This Buddhist mudra is called "The Gesture of Debate" or "discussion" mudra. It is a mystic gesture of Taras and Bodhisattvas in which tips of thumb and index finger touch forming a circle, and all the other fingers are extended upwards. The Mudra is? a) Vitarka b) Abhaya c) Varada d) Shramanamudra Solution: a)

Learning: Option A: Vitarka Mudra.

INSIGHTS PRELIMS TEST SERIES 2017 www.insightsonindia.com www.insightsias.com TEST – 5 Solutions

Option B: This gesture is also called "Gesture of Fearlessness" or "blessing" or "fearless mudra". Generally, this position is shown with the palms(s) facing outward and the fingers extended upwards. This mudra is characteristic of Buddha Shakyamuni and Dhyani Buddha Amogasiddhi.

Option C: This gesture is also called "Gift bestowing Gesture of Compassion" or "conferring boon" or "grace" mudra. The arm is extended all way down with palm facing outwards.

Option D: This gesture is also called renunciation mudra. The hand points downward away from the body as a symbol for renunciation of secular pleasures.

Q Source: Improvisation: Page 104: Introduction to Indian Art – Class XI

17. Which of the following will violate the General Anti Avoidance Rules (GAAR) in India? 1. Any speculative activity in the security markets

INSIGHTS PRELIMS TEST SERIES 2017 www.insightsonindia.com www.insightsias.com TEST – 5 Solutions

2. Any real estate deal that has been settled in a short period of time

Which of the above is/are correct?

a) 1 only b) 2 only c) Both 1 and 2 d) None Solution: d)

Justification: About GAAR: GAAR provisions are targeted at arrangement or transactions made specifically to avoid taxes.

Evolution: In 2007, Vodafone entered the Indian market by buying Hutchison Essar. The deal took place in Cayman Islands. The Indian government claimed over US$2 billion were lost in taxes.

Later, a notice was sent to Vodafone. Vodafone claimed that the transaction was not taxable as it was between two foreign firms. The government claimed that the deal was taxable as the underlying assets involved were located in India. Then came the provisions of GAAR in India.

Provisions: The regulation allows tax officials to deny tax benefits, if a deal is found without any commercial purpose other than tax avoidance.

• It allows tax officials to target participatory notes.

• Under GAAR, the investor has to prove that the participatory note was not set to avoid taxes.

• It also allows officials to deny double taxation avoidance benefits, if deals made in tax havens like Mauritius were found to be avoiding taxes

Statement 1: Therefore, clearly it does not apply to all such transactions. Only if the speculation was done in order to specifically avoid taxes, it will come under the radar of GAAR.

INSIGHTS PRELIMS TEST SERIES 2017 www.insightsonindia.com www.insightsias.com TEST – 5 Solutions

Statement 2: The same applies to real estate transactions. The time period of sale/purchase is not the issue; it is the purpose with which it is done matters.

Q Source: Q35: SET A: CAPF 2014

18. Consider the following about the Nataraja dancing posture depicted in Chola period bronze sculpture. 1. The dance is associated with the end of the cosmic world. 2. Shiva has been shown balancing himself on his left leg and suppressing the demon of ignorance. 3. Shiva’s main right hand is posed in the bhumisparsha hasta. 4. Tirobhava depicted in the sculpture shows the birth of the Universe.

Select the correct answer using the codes below.

a) 1 and 3 only b) 2 and 4 only c) 1, 2 and 3 only d) 1 only Solution: d)

Justification: Statement 2 and 4: UPSC often plays with words, for e.g. 100 instead of 10, right instead of left, up instead of down etc. You should be prepared for everything.

In the bronze sculpture Shiva has been shown balancing himself on his “right” leg and suppressing the apasmara, the demon of ignorance or forgetfulness, with the foot of the same leg.

At the same time he raises his left leg in bhujangatrasita stance, which represents tirobhava, that is kicking away the veil of “maya or illusion” from the devotee’s mind.

INSIGHTS PRELIMS TEST SERIES 2017 www.insightsonindia.com www.insightsias.com TEST – 5 Solutions

Statement 3: His four arms are outstretched and the main right hand is posed in abhaya hasta or the gesture suggesting.

The upper right holds the damaru his favourite musical instrument to keep on the beat tala.

Q Source: Page 107: Introduction to Indian Art – Class XI

19. The Roaring Forties are a) Strong westerly winds found in the Southern Hemisphere b) Feeble easterly winds found in the Northern Hemisphere c) Thunderstorms often found near the equator d) Equatorial counter-currents that move near the eastern coastal shores in lower latitudes Solution: a)

INSIGHTS PRELIMS TEST SERIES 2017 www.insightsonindia.com www.insightsias.com TEST – 5 Solutions

Learning: They are generally found between the latitudes of 40 and 50 degrees.

• The strong west-to-east air currents are caused by the combination of air being displaced from the Equator towards the South Pole and the Earth's rotation, and there are few landmasses to serve as windbreaks.

• The Roaring Forties were a major aid to ships sailing the Brouwer Route from Europe to the East Indies or Australasia during the Age of Sail, and in modern usage are favoured by yachtsmen on round- the-world voyages and competitions.

• The boundaries of the Roaring Forties are not consistent, and shift north or south depending on the season.

• Similar but stronger conditions occur in more southerly latitudes and are referred to as the Furious Fifties and Shrieking or Screaming Sixties.

Q Source: Q7: SET A: CAPF 2014

20. Which of the following most appropriately describes “Toranas”? a) Calligraphy inscribed on interior walls of royal durbars b) Style of dome-based architecture introduced by Turks c) Collection of Chinars and Pietra duras seen in decorative works in Islami architecture d) Gateway to Buddhist Stupas Solution: d)

Learning: Usually made of stone, Toranas mark the entrance to a Buddhist shrine or stupa or to a Hindu temple.

Toranas typically consist of two pillars carrying two or three transverse beams that extend beyond the pillars on either side.

INSIGHTS PRELIMS TEST SERIES 2017 www.insightsonindia.com www.insightsias.com TEST – 5 Solutions

Strongly similar to wooden construction, toranas are often covered from top to bottom with exquisite sculpture.

The four toranas of the Great Stupa at Sanchi are relevant examples.

Q Source: Improvisation: Page 111: Introduction to Indian Art – Class XI

21. With reference to art and archaeological history of India, which of the following are associated with the reign of Ashoka? 1. Rock-cut Elephant at Dhauli 2. Lomus Rishi caves 3. Badami Caves 4. Udayagiri Caves

Select the correct answer using the codes below.

a) 1 and 2 only b) 3 and 4 only c) 1 and 3 only d) 2 and 4 only Solution: a)

Justification: Statement 1: Rock-cut Elephant at Dhauli was created during Ashoka’s reign (272-231 BC). It is the earliest Buddhist sculpture of Odisha.

Statement 2: Located in the Barabar and Nagarjuni hills of Jehanabad district in Bihar, this rock-cut cave was carved out as a sanctuary. It was built during the Ashokan period of the Maurya Empire in the 3rd century BC, as part of the sacred architecture of the Ajivikas.

Statement 3: The Badami cave temples are a complex of Hindu and Jain cave temples located in Bagalkot district in northern part of , India. The caves are considered an example of Indian rock-cut

INSIGHTS PRELIMS TEST SERIES 2017 www.insightsonindia.com www.insightsias.com TEST – 5 Solutions architecture, especially Badami Chalukya architecture, which dates from the 6th century.

Statement 4: It is a Hindu cave and was made during the Gupta reign. It contains the famous Varaha sculpture.

Q Source: Improvisation: Page 21: Introduction to Indian Art – Class XI

22. Internal and Extra Budgetary Resources (IEBR) was recently in news. What is IBER? a) Financing received from IFC Masala bonds b) Basel III Capitalization of Commercial Banks c) Monetization of deficit through Forex Reserves d) Part of Central plan constituting resources raised by the PSUs through profits, loans and equity. Solution: d)

Learning: IEBR is an important part of the Central plan of the Government of India and constitutes the resources raised by the PSUs through profits, loans and equity.

• The Union Cabinet has recently given approval to service principal and interest against the Extra Budgetary Resources (EBR) of Rs. 16,300 crore by Union Government for infrastructure spending.

• The EBR is proposed to finance funds to be raised by Indian Renewable Energy Development Agency (IREDA), Power Finance Corporation (PFC), Inland Waterways Authority of India (IWAI), and National Bank for Agriculture and Rural Development (NABARD).

INSIGHTS PRELIMS TEST SERIES 2017 www.insightsonindia.com www.insightsias.com TEST – 5 Solutions

• This implies that interest and principal in respect of the EBR of Rs.16,300 crore will be financed by Union Government by making suitable budget provisions.

• It will supplement the efforts of the Union Government to improve infrastructure spending and also improve revenue-capital mix of expenditure for a more sustainable growth.

Q Source: http://pib.nic.in/newsite/PrintRelease.aspx?relid=150969

23. Consider the following about Ken-Betwa river interlinking project. 1. It will be the first river project that will be located within a tiger reserve. 2. It will help irrigate the highly drought-affected and drought- prone region of Bundelkhand.

Which of the above is/are correct?

a) 1 only b) 2 only c) Both 1 and 2 d) None Solution: c)

Justification: Statement 1: The National Board for Wildlife (NBWL) has given its approval to the Ken-Betwa river interlinking project. It will submerge about 10% of the Panna Tiger Reserve in Madhya Pradesh which has been feted as a model tiger-conservation reserve.

Statement 2: Apart from Bundelkhand region, it will benefit 5 districts of Madhya Pradesh (Chhatarpur, Raisen, Panna,Tikamgarh, and Vidisha) by assured irrigation supply, domestic and industrial water supply and power.

INSIGHTS PRELIMS TEST SERIES 2017 www.insightsonindia.com www.insightsias.com TEST – 5 Solutions

On the other hand, about 20000 people in 38 villages will be affected due to the submergence by Daudhan reservoir and Makodia reservoir.

Q Source: http://indianexpress.com/article/india/india-news- india/ken-betwa-river-linking-project-wildlife-board-panel-gives-nod- for-phase-i-3040084/

24. The International Day of Peace is also the 1. Founding date of the United Nations 2. Date when the United Nations General Assembly was instituted

Which of the above is/are correct?

a) 1 only b) 2 only c) Both 1 and 2 d) None Solution: d)

Justification: The International Day of Peace is observed across the world on 21 September every year

Statement 1: The United Nations General Assembly (UNGA) had designated 21 September as the International Day of Peace in 1981. So, 1 is clearly wrong.

Statement 2: This day coincides with its opening session of UNGA which usually held annually on the third Tuesday of September.

2016 Theme is: “The Sustainable Development Goals: Building Blocks for Peace”.

Q Source: 21st September: International Day of Peace

INSIGHTS PRELIMS TEST SERIES 2017 www.insightsonindia.com www.insightsias.com TEST – 5 Solutions

25. Persistent Organic Pollutants (POPs) are toxic organic chemical substances show which of the following properties? 1. Accumulate in the water component of the bodies of living organisms and thus spread to higher trophic levels 2. Found at higher concentrations at higher levels in the food chain

Which of the above is/are correct?

a) 1 only b) 2 only c) Both 1 and 2 d) None Solution: b)

Justification: They are carbon-based compounds and possess a particular combination of physical and chemical properties such that, once released into the environment, they:

• remain intact for exceptionally long periods of time (many years); • become widely distributed throughout the environment as a result of natural processes involving soil, water and, most notably, air; • accumulate in the fatty tissue (not water tissues, so 1 is wrong) of living organisms including humans, and are found at higher concentrations at higher levels in the food chain; and • are toxic to both humans and wildlife. Learning: As a result of releases to the environment over the past several decades due especially to human activities, POPs are now widely distributed over large regions (including those where POPs have never been used) and, in some cases, they are found around the globe.

Stockholm convention deals with these POPs on a global scale. India is a signatory to the convention

Q Source: http://chm.pops.int/TheConvention/Overview/tabid/3351/Default.aspx

INSIGHTS PRELIMS TEST SERIES 2017 www.insightsonindia.com www.insightsias.com TEST – 5 Solutions

26. Mrs Hume's Pheasant is found in its natural habitat in a) Western Indian Desert b) Ladakh region c) Wetlands of Uttar Pradesh d) North-eastern India Solution: d)

Learning: In India, it occurs in Manipur, Mizoram, Nagaland and perhaps Arunachal Pradesh in north-east India. As per Birdlife International, “It appears to be a rare species in India”. It is the state bird of Manipur.

It inhabits open, dry, subtropical evergreen (mainly oak), coniferous (chiefly pine) or mixed conifer-broadleaf forests on steep, often rocky hillsides interrupted by scrub and grassy clearings.

The ease with which it can be trapped has been a major cause of its continuing decline across much of its range, including populations within protected areas.

The species is legally protected in India, Thailand, Myanmar and China.

Q Source: Syllabus: State birds, animals, trees and flowers

27. Newlands’ Law of Octaves is related to a) Composition of Music b) Classification of elements c) Changing phases of matter d) Distribution of mass in the Solar system Solution: b)

INSIGHTS PRELIMS TEST SERIES 2017 www.insightsonindia.com www.insightsias.com TEST – 5 Solutions

Note: We are venturing into topics/areas hitherto untouched by conventional thinking, and by UPSC too. You will find many questions that may not seem very important from exam point of view; nevertheless we should be covering them because in UPSC there are no trends. You should be prepared for anything.

Justification: John Newlands, an English scientist, arranged the then known elements in the order of increasing atomic masses. He started with the element having the lowest atomic mass (hydrogen) and ended at thorium which was the 56th element.

He found that every eighth element had properties similar to that of the first. He compared this to the octaves found in music.

Therefore, he called it the ‘Law of Octaves’. It is known as ‘Newlands’ Law of Octaves’. In Newlands’ Octaves, the properties of lithium and sodium were found to be the same. Sodium is the eighth element after lithium. Similarly, beryllium and magnesium resemble each other.

Q Source: Page 80: Science Xth Standard NCERT

28. Consider the following about The Energy and Resources Institute (TERI). 1. It is a research arm of the Ministry of Environment, Forests and Climate Change (MoEF&CC). 2. Oilzapper technology for bio-remediation of oily sludge and oil spills was developed by TERI.

Which of the above is/are correct?

a) 1 only b) 2 only c) Both 1 and 2 d) None Solution: b)

INSIGHTS PRELIMS TEST SERIES 2017 www.insightsonindia.com www.insightsias.com TEST – 5 Solutions

Justification: Statement 1: It is a leading think tank dedicated to conducting research for sustainable development of India and the Global South.

TERI was established in 1974 as an information centre on energy issues.

Statement 2: Nearly 70% of oil refineries in India use the Oilzapper technology developed by TERI.

• Oilzapper, a bacterial consortium that degrades crude oil and oil sludge, has reclaimed thousands of hectares of contaminated cropland in different parts of India. • TERI is now cleaning up nearly 2,40,000 sq meters of oil spills in Kuwait-the first-of-its-kind large-scale bioremediation project implemented by India's biotechnology sector.

Learning: Moreover, Green Rating for Integrated Habitat Assessment (GRIHA) was conceived by TERI and developed with Ministry of New and Renewable Energy. It is a national rating system for green buildings in India.

TERI's mycorhizzal technology has made its mark as a commercially successful bio-fertilizer in India, Europe and North America.

It is being used by large industries to reclaim toxic wastelands; and at the same time helping to boost crop yields by nearly 25% thus significantly curtailing the use of chemical fertilizers.

Q Source: Often in news + Improvisation: http://www.moef.nic.in/sites/default/files/treaties/international- treaties.html

29. The first discovery of rock paintings was made in India in 1867-68 by a) Richard J. C. Atkinson

INSIGHTS PRELIMS TEST SERIES 2017 www.insightsonindia.com www.insightsias.com TEST – 5 Solutions

b) William F. Albright c) Max Muller d) Archibold Carlleyle Solution: d)

Learning: It is interesting to know that the first discovery of rock paintings was made in India in by the archaeologist twelve years before the discovery of Altamira in Spain.

Cockburn, Anderson, Mitra and Ghosh were the early archaeologists who discovered a large number of sites in the Indian sub-continent.

Remnants of rock paintings have been found on the walls of the caves situated in several districts of Madhya Pradesh, Uttar Pradesh, Andhra Pradesh, Karnataka and Bihar. Some paintings have been reported from the Kumaon hills in Uttarakhand also.

Q Source: Page 2: Introduction to Indian Art – Class XI

30. Which of the following is/are plant hormones? 1. Thyroxin 2. Gibrellin 3. Cytokinin 4. Oestrogen

Select the correct answer using the codes below.

a) 1 and 4 only b) 2 and 3 only c) 3 only d) 1, 2, 3 and 4 Solution: b)

INSIGHTS PRELIMS TEST SERIES 2017 www.insightsonindia.com www.insightsias.com TEST – 5 Solutions

Justification: Statement 2: Gibberellins (GAs) are plant hormones that regulate growth and influence various developmental processes, including stem elongation, germination, dormancy, flowering, sex expression, enzyme induction, and leaf and fruit senescence.

Statement 3: They promote cell division, or cytokinesis, in plant roots and shoots.

Learning: Abscisic acid is one example of a hormone which inhibits growth. Its effects include wilting of leaves.

Auxin also promotes plant growth.

Q Source: Page 122: Science Xth Standard NCERT

31. Lima Declaration is related to a) Protection of biodiversity in ‘global hotspots’ b) Mitigation of Green House Gases (GHGs) under supplementary guidelines to the Kyoto Protocol c) Redistribution of world industry towards developing countries d) Empowering women to break the “Glass Ceiling” in the leadership of the highest global institutions Solution: c)

Learning: The Lima Declaration and Plan of Action calls for the redistribution of world industry so that developing countries would have 25% of it by the year 2000. To achieve this, radical changes in traditional concepts and practices are recommended.

• To facilitate this, it was recommended that UNIDO become a specialised agency of the United Nations, with a new Industrial Development Fund, and undertake the central co-ordinating role in changing the world industrial map.

INSIGHTS PRELIMS TEST SERIES 2017 www.insightsonindia.com www.insightsias.com TEST – 5 Solutions

• The Lima Declaration calls upon the developed countries to eliminate barriers to trade with developing countries and encourage their manufactured exports. • They are asked to "restructure" their industries in order to deploy production capacity to developing countries and to expand technical assistance programmes.

Q Source: Improvisation: https://www.theguardian.com/commentisfree/2016/sep/14/agenda-21- is-conspiracy-theory-but-dont-dismiss-malcolm-roberts-as-a-harmless- kook + You can also find the declaration mentioned quite often in Economic Surveys of India

32. Power of a spherical lens is mainly dependent upon its 1. Length 2. Width 3. Focal length 4. Clarity

Select the correct answer using the codes below.

a) 2 and 4 only b) 1, 2 and 4 only c) 3 only d) 1, 2, 3 and 4 Solution: c)

Justification: Two lenses with same focal length can have different lengths, width and clarity; yet their power will remain the same. So, 1, 2 and 4 are incorrect.

Power of lens is the ability of the lens to converge the rays of light falling on it. Definition: The power of a lens is the reciprocal of its focal length in meters. When, f = 1 m, P = 1/f = 1/1 = 1 dioptre. 1 dioptre is the power of a lens whose focal length is 1 meter.

Q Source: Page 184: Science Xth Standard NCERT

INSIGHTS PRELIMS TEST SERIES 2017 www.insightsonindia.com www.insightsias.com TEST – 5 Solutions

33. Which of these tribes can be found in the Eastern Himalayan region? 1. Rabhas 2. Hajongs 3. Khamti 4. Chakmas

Select the correct answer using the codes below.

a) 1, 2 and 3 only b) 3 and 4 only c) 1 and 2 only d) 1, 2, 3 and 4 Solution: d)

Justification: Statement 1: Rabha is a Scheduled Tribe community of Assam, Meghalaya and West Bengal. The language/dialect spoken by the Rabha people is also of the same name.

The Rabhas belong to the Indo Mongoloid group of people and have similarities with other members of Koch group such as Garos.

Statement 2: They are one of the largest ethnicities in Meghalaya. Hajong people are spread out across North East India West Bengal and Bangladesh.

Hajongs are predominantly rice farmers. Hajong have the status of a Scheduled Tribe in India.

Statement 3: They can be found in northwestern Burma as well as Lohit district of Arunachal Pradesh in India. Smaller numbers can be found in parts of Assam.

INSIGHTS PRELIMS TEST SERIES 2017 www.insightsonindia.com www.insightsias.com TEST – 5 Solutions

The Tai-Khamti are followers of Theravada Buddhism.

Statement 4: Chakmas are spread across Bangladesh and parts of northeastern India, western Burma etc.

They have their own language, customs and culture, and profess Theravada Buddhism.

Q Source: Syllabus: Eastern Himalayan Biodiversity

34. Food and Agriculture Organization (FAO) of the United Nations estimates that 25% of the world's food crops are affected by mycotoxins. The most notorious of these are aflatoxins. They are produced by? a) Protozoa b) Yeast c) Bacteria d) Fungi Solution: d)

Justification: Molds are microscopic fungi that live on plant or animal matter.

• Mycotoxins are poisonous substances produced by certain molds found primarily in grain and nut crops, but are also known to be on celery, grape juice, apples, and other produce. • Mycotoxins can penetrate into the parts of food that are not visibly mouldy as well. It is therefore necessary to throw away all of the food if any part of it is mouldy. They are also notoriously difficult to destroy as they are stable to both heat and chemicals.

Learning: Aflatoxin is (poison) any of a family of mycotoxins, produced by molds of the aspergillus genus.

INSIGHTS PRELIMS TEST SERIES 2017 www.insightsonindia.com www.insightsias.com TEST – 5 Solutions

• Aflatoxins have been associated with various diseases, such as aflatoxicosis in livestock, domestic animals, and humans throughout the world. • Patulin is a mycotoxin produced by a variety of molds, in particular, Aspergillus and Penicillium and Byssochlamys. Most commonly found in rotting apples, in general the amount of patulin in apple products is viewed as a measure of the quality of the apples used in production. • Aspergillus is used in the commercial production of citric acid. Q Source: Syllabus: Important microorganisms – Bacteria, Fungi, Virus etc.

35. Consider the following statements.

Assertion (A): The apparent position of the star on earth is slightly different from its actual position.

Reason (R): The starlight, on entering the earth’s atmosphere, undergoes refraction continuously before it reaches the earth.

In the context of the above, which of these is correct?

a) A is correct, and R is an appropriate explanation of A. b) A is correct, but R is not an appropriate explanation of A. c) A is correct, but R is incorrect. d) Both A and R are incorrect. Solution: a)

Justification: The atmospheric refraction occurs in a medium of gradually changing refractive index. Since the atmosphere bends starlight towards the normal, the apparent position of the star is slightly different from its actual position.

INSIGHTS PRELIMS TEST SERIES 2017 www.insightsonindia.com www.insightsias.com TEST – 5 Solutions

• The star appears slightly higher (above) than its actual position when viewed near the horizon.

• Further, this apparent position of the star is not stationary, but keeps on changing slightly, since the physical conditions of the earth’s atmosphere are not stationary.

• Since the stars are very distant, they approximate point-sized sources of light.

• As the path of rays of light coming from the star goes on varying slightly, the apparent position of the star fluctuates and the amount of starlight entering the eye flickers – the star sometimes appears brighter, and at some other time, fainter, which is the twinkling effect

Q Source: Page 195: Science Xth Standard NCERT

36. Consider the following statements.

Assertion (A): Viruses found in food cannot infect humans.

Reason (R): Unlike Bacteria, Viruses cannot multiply in foods.

In the context of the above, which of these is correct?

a) A is correct, and R is an appropriate explanation of A. b) A is correct, but R is not an appropriate explanation of A. c) A is correct, but R is incorrect. d) A is incorrect, but R is correct. Solution: d)

Justification: Viruses in food do affect humans, but Viruses need live host for multiplication. They can only multiply inside living cells of other organisms. They do not multiply in food items, so food only acts as a mechanism of transmission.

INSIGHTS PRELIMS TEST SERIES 2017 www.insightsonindia.com www.insightsias.com TEST – 5 Solutions

The main modes of transmission of Virus are by food handlers and the use of dirty utensils, which transfer the virus to food whereupon it is ingested by humans.

Q Source: Syllabus: Important microorganisms – Bacteria, Fungi, Virus etc.

37. Consider the following about Indian rose chestnut, also called as cobra's saffron. 1. It is a tree species found in of India. 2. It is the smallest tree found in the hilly regions of India. 3. The tree is considered auspicious in many regions of India as it attains maturity within a full moon night.

Select the correct answer using the codes below.

a) 1 and 2 only b) 2 and 3 only c) 1 only d) 1 and 3 only Solution: c)

Justification: Statement 1: It is widely cultivated as an ornamental due to its leaves and fragrant white flowers Nageshwar (the state flower of Tripura).

It is native to wet, tropical parts of Sri Lanka, India, southern Nepal, Burma, Thailand, Indochina, the Philippines, Malaysia and Sumatra, where it grows in evergreen forests, especially in river valleys.

In the eastern Himalayas and Western Ghats in India it grows up to altitudes of 1,500 m, while in Sri Lanka up to 1,000 m.

INSIGHTS PRELIMS TEST SERIES 2017 www.insightsonindia.com www.insightsias.com TEST – 5 Solutions

Statement 2: The tree can grow over 30 m tall, often buttressed at the base with a trunk up to 2 m in diameter.

Statement 3: It is a slow growing tree.

Q Source: Syllabus: State birds, animals, trees and flowers

38. Which of the following is/are benefits of Probiotics? 1. They increase the pH of the stomach making digestion easier. 2. They make diarrheal diseases milder. 3. They kill major bacterial strains in the human gut improving bowel movement.

Select the correct answer using the codes below.

a) 1 and 2 only b) 3 only c) 2 only d) 1 and 3 only Solution: c)

Justification: They are live food supplements used in yoghurt and other fermented milk products.

It includes Lactobacillus acidophillus and Bifidobacterium bifidum. A minimum of 10^8 bacteria per 1 ml must get to the colon alive to have any significant effect.

These bacteria improve the microbial spectrum in the gut and thus contribute to the following effects:

• Influence immunity and hence prevent or make diarrheal diseases milder • Decrease the risk of colon cancer • Decrease cholesterol absorption

INSIGHTS PRELIMS TEST SERIES 2017 www.insightsonindia.com www.insightsias.com TEST – 5 Solutions

• Produce acids that decrease the pH in the gut and thus increase the absorption of minerals such as calcium and phosphorous.

Q Source: Syllabus: Important microorganisms – Bacteria, Fungi, Virus etc.

39. In India, the largest contributor to energy generation is a) Coal-based power plants b) Nuclear plants c) Natural gas based plants d) Hydel power plants Solution: a)

Learning: India became the world's third largest producer of electricity in the year 2013 with 4.8% global share in electricity generation surpassing Japan and Russia.

India's electricity sector consumes about 72% of the coal produced in the country. The high ash content in India's coal affects the thermal power plant's potential emissions.

INSIGHTS PRELIMS TEST SERIES 2017 www.insightsonindia.com www.insightsias.com TEST – 5 Solutions

Q Source: Page 244: Science Xth Standard NCERT

40. In recent years, the population of Syrus crane has declined due to 1. Loss of wetlands and conversion of land for agriculture 2. Collision of these birds with power lines 3. Ingesting pesticides while foraging in the crop fields

Select the correct answer using the codes below.

a) 1 and 2 only b) 3 only c) 1 only d) 1, 2 and 3 Solution: d)

Justification: The Sarus crane is the tallest flying bird in the world. Nests are constructed on water in natural wetlands or in flooded paddy fields.

Statement 1: The main threat to the Sarus crane in India is habitat loss and degradation due to draining the wetland and conversion of land for agriculture. The landscape of its historic range is rapidly changing due to construction of highways, housing colonies, roads, and railway lines.

Statement 2 and 3: More recently, many deaths have been recorded due to collision with power lines. Also, due to the increase in agricultural land, Sarus cranes are left with no choice but to forage in these fields, and as a result ingest pesticides, which lead to poisoning.

WWF-India has provided technical support for the restoration and management of key wetlands

INSIGHTS PRELIMS TEST SERIES 2017 www.insightsonindia.com www.insightsias.com TEST – 5 Solutions

Q Source: Syllabus: State birds, animals, trees and flowers

41. Which of the following human body fluids is the most acidic in nature? a) Blood b) Gastric Juice c) Saliva d) Tears Solution: b)

Learning:

Q Source: Page 26: Science Xth Standard NCERT

INSIGHTS PRELIMS TEST SERIES 2017 www.insightsonindia.com www.insightsias.com TEST – 5 Solutions

42. The practice of Vegetative Propagation is beneficial because 1. It can help propagate even those plants that have lost the capacity to produce seeds. 2. Plants produced under the practice bear no genetic resemblance to the parent plant thus promoting biodiversity.

Which of the above is/are correct?

a) 1 only b) 2 only c) Both 1 and 2 d) None Solution: a)

Justification & Learning: There are many plants in which parts like the root, stem and leaves develop into new plants under appropriate conditions.

• Unlike in most animals, plants can indeed use such a mode for reproduction. This property of vegetative propagation is used in methods such as layering or grafting to grow many plants like sugarcane, roses, or grapes for agricultural purposes.

• Plants raised by vegetative propagation can bear flowers and fruits earlier than those produced from seeds. Such methods also make possible the propagation of plants such as banana, orange, rose and jasmine that have lost the capacity to produce seeds. So, 1 is correct.

• Another advantage of vegetative propagation is that all plants produced are genetically similar enough to the parent plant to have all its characteristics. So, 2 is wrong.

Q Source: Page 131: Science Xth Standard NCERT

INSIGHTS PRELIMS TEST SERIES 2017 www.insightsonindia.com www.insightsias.com TEST – 5 Solutions

43. Consider the following statements. 1. The Shramana tradition, based on the principle of Ahimsa, was an essential component of the ancient Vedic religion. 2. The Ajivika School believes in the doctrine of absolute determinism arguing there is no free will.

Which of the above is/are correct?

a) 1 only b) 2 only c) Both 1 and 2 d) None Solution: b)

Justification: Statement 1: The term refers to several Indian religious movements parallel to but separate from the historical Vedic religion. The Sramaṇa tradition includes Jainism of 9th-century BCE, Buddhism of 6th-century BCE, and others such as Ajivika, Ajnana and Carvaka.

Statement 2: The Ajivika school is known for its Niyati doctrine of absolute determinism, the premise that there is no free will, that everything that has happened, is happening and will happen is entirely preordained and a function of cosmic principles.

Aivika considered the karma doctrine as a fallacy.

Ajivika metaphysics included a theory of atoms similar to the Vaisheshika school, where everything was composed of atoms, qualities emerged from aggregates of atoms, but the aggregation and nature of these atoms was predetermined by cosmic forces.

Q Source: Improvisation: Page 19: Introduction to Indian Art – Class XI

44. Consider the map that shows the distribution of a particular species found in India.

INSIGHTS PRELIMS TEST SERIES 2017 www.insightsonindia.com www.insightsias.com TEST – 5 Solutions

The shaded area in the map is the natural habitat of?

a) Emerald dove b) Great Indian hornbill c) Indian roller d) Black necked crane Solution: b)

Learning: They occur in primary evergreen and moist deciduous forest, mainly in lowland forest, but they can be found up to 2,000 meters in some areas.

In north-east India, they are generally not recorded above 1000 m. They are also seen in selectively logged forests and plantations close to larger forests.

They are covered under Schedule I of the Wildlife Protection Act of India and have been declared the state bird of Arunachal Pradesh.

Indian Protected Areas where they are seen commonly are the Anamalai Tiger Reserve and Dandeli National Park (Western Ghats), Corbett Tiger Reserve (north India) and Kaziranga, Pakke, Namdapha and Manas (north-east India).

Q Source: Syllabus: State birds, animals, trees and flowers

INSIGHTS PRELIMS TEST SERIES 2017 www.insightsonindia.com www.insightsias.com TEST – 5 Solutions

45. Litmus solution is a purple dye, which is extracted from “X”. “X” is a plant belonging to the division Thallophyta, and is commonly used as an indicator. X here refers to? a) Lichens b) Mosses c) Shell d) Rosua Solution: a)

Learning: Litmus is a water soluble mixture of different dyes extracted from lichens. It is often absorbed onto filter paper to produce one of the oldest forms of pH indicator, used to test materials for acidity.

When the litmus solution is neither acidic nor basic, its colour is purple.

There are many other natural materials like red cabbage leaves, turmeric, coloured petals of some flowers such as Hydrangea, Petunia and Geranium, which indicate the presence of acid or base in a solution.

Q Source: Page 17: Science Xth Standard NCERT

46. Consider the following about Brahma Kamal that is native to the Himalayas and Uttarakhand. 1. It blooms during Monsoon season and perishes before the winter season. 2. A postal stamp has been issued by the Indian Postal Department to commemorate this flower.

Which of the above is/are correct?

a) 1 only

INSIGHTS PRELIMS TEST SERIES 2017 www.insightsonindia.com www.insightsias.com TEST – 5 Solutions

b) 2 only c) Both 1 and 2 d) None Solution: c)

Justification: Statement 1: Flowers bloom in mid-monsoon (July– August) at an altitudinal range of 3000–4800 m.

The flowers can be seen till mid-October, after which the plant perishes, becoming visible again in April.

According to folklore in Hindu mythology, Brahma Kamal was created by Lord Brahma to help Lord Shiva place the head of an elephant on the body of Lord Ganesha. The flower dropped ‘Amruta’ – the elixir of life from its petals on the body.

Statement 2: It is the state flower of Uttarakhand.

This plant is applied to cuts and bruises as local medicine. Brahma Kamal is a herb in Tibetan medicine. It has a somewhat bitter taste and it promotes heat.

INSIGHTS PRELIMS TEST SERIES 2017 www.insightsonindia.com www.insightsias.com TEST – 5 Solutions

Q Source: Syllabus: State birds, animals, trees and flowers

47. Consider the following statements.

Assertion (A): The sky appears dark to passengers flying at very high altitudes.

Reason (R): Light scattering is not prominent at such heights.

In the context of the above, which of these is correct?

a) A is correct, and R is an appropriate explanation of A. b) A is correct, but R is not an appropriate explanation of A. c) A is correct, but R is incorrect. d) Both A and R are incorrect. Solution: a)

Justification: The molecules of air and other fine particles in the atmosphere have size smaller than the wavelength of visible light. These are more effective in scattering light of shorter wavelengths at the blue end than light of longer wavelengths at the red end.

The red light has a wavelength about 1.8 times greater than blue light. Thus, when sunlight passes through the atmosphere, the fine particles in air scatter the blue colour (shorter wavelengths) more strongly than red.

The scattered blue light enters our eyes. If the earth had no atmosphere, there would not have been any scattering. Then, the sky would have looked dark. The sky appears dark to passengers flying at very high altitudes, as scattering is not prominent at such heights.

Q Source: Page 196: Science Xth Standard NCERT

INSIGHTS PRELIMS TEST SERIES 2017 www.insightsonindia.com www.insightsias.com TEST – 5 Solutions

48. In India, which of these matters comes under the State List of the Seventh Schedule to the Indian Constitution? a) Education b) Forests c) Irrigation d) Banking Solution: c)

Learning: Union List includes subjects of national importance such as defence of the country, foreign affairs, banking, communications and currency.

State List contains subjects of State and local importance such as police, trade, commerce, agriculture and irrigation.

Concurrent List includes subjects of common interest to both the Union Government as well as the State Governments, such as education, forest, trade unions, marriage, adoption and succession. Both the Union as well as the State Governments can make laws on the subjects mentioned in this list.

Q Source: Page 16: Democratic Politics: Class Xth NCERT

49. What are the general applications of carbon Nanotubes? 1. They can be used to reinforce graphite in tennis rackets. 2. Their structure allows them to be used as a container for transporting a drug in the body. 3. They are used as semiconductors in electrical circuits.

Select the correct answer using the codes below.

INSIGHTS PRELIMS TEST SERIES 2017 www.insightsonindia.com www.insightsias.com TEST – 5 Solutions

a) 1 and 3 only b) 2 only c) 1 and 2 only d) 1, 2 and 3 Solution: d)

Justification: The fullerenes are a large class of allotropes of carbon and are made of balls, ‘cages’ or tubes of carbon atoms.

Statement 1: Nanotubes are cylindrical fullerenes. Their unique molecular structure results in extraordinary macroscopic properties, including high tensile strength, high electrical conductivity, high ductility, high heat conductivity, and relative chemical inactivity.

Nanotubes can be used to reinforce graphite in tennis rackets because they are very strong. Due to the above mentioned properties, they are also used as semiconductors in electrical circuits.

Statement 3: The nanotube's structure allows it to be used as a container for transporting a drug in the body. A molecule of the drug can be placed inside the nanotube cage. This keeps the drug 'wrapped up' until it reaches the site where it is needed. In this way, a dose that might be damaging to other parts of the body can be delivered safely to, for example, a tumour.

Q Source: Improvisation: Page 61: Science Xth Standard NCERT

50. Consider the following matches of sites of pre-historic rock paintings with the state they are located in. 1. Lakhudiyar : Odisha 2. Kaimurean extensions : Jammu & Kashmir 3. Bhimbetka : Madhya Pradesh 4. Kapugallu : Telangana

Select the correct answer using the codes below.

INSIGHTS PRELIMS TEST SERIES 2017 www.insightsonindia.com www.insightsias.com TEST – 5 Solutions

a) 1 and 3 only b) 2, 3 and 4 only c) 3 and 4 only d) 1 and 2 only Solution: c)

Justification: Statement 1: The rock shelters on banks of the River Suyal at Lakhudiyar (Uttarakhand), about twenty kilometres on the Almora–Barechina road, bear these prehistoric paintings. Lakhudiyar literally means one lakh caves.

Statement 2: Richest paintings are reported from the Vindhya ranges of Madhya Pradesh and their Kaimurean extensions into Uttar Pradesh. These hill ranges are full of Palaeolithic and Mesolithic remains.

Statement 4: The granite rocks of Karnataka, Andhra Pradesh and provided suitable canvases to the Neolithic paintings. There are several such sites but more famous among them are Kupgallu (Telangana), Piklihal and Tekkalkota.

Bhimbetkas has been covered in an earlier Test.

Q Source: Page 2: Introduction to Indian Art – Class XI

51. Consider the following about The Incredible India Tourism Investors Summit (IITIS) 2016. 1. IITIS will provide a common platform for domestic and international investors to meet project owners from across Indian states and the private sector. 2. IITIS has been convened with a listed objective to raise funding for the “HRIDAY” scheme.

Which of the above is/are correct?

a) 1 only b) 2 only

INSIGHTS PRELIMS TEST SERIES 2017 www.insightsonindia.com www.insightsias.com TEST – 5 Solutions

c) Both 1 and 2 d) None Solution: a)

Justification: Statement 1: It will host presentations of investible properties, session on why invest in India, Micro of Small and Micro Enterprises (MSME) in tourism sector.

There will be participation from major stakeholders including states/UTs with their ready investible projects, banks and financial Institutions, business developers.

Statement 2: Besides, it will also host panel discussions on core infrastructure for tourism, session on start-ups, digital India, Swadesh Darshan, PRASAD and niche tourism products, B2B meetings.

Q Source: IITIS 2016 Summit

52. The Indus Valley civilization Sites, among the following, that belong to India are 1. Harappa 2. Lothal 3. Dholavira 4. Mohenjodaro 5. Kalibangan

Select the correct answer using the codes below.

a) 1, 2 and 4 only b) 3, 4 and 5 only c) 2, 3 and 5 only d) 1, 2, 3, 4 and 5 Solution: c)

INSIGHTS PRELIMS TEST SERIES 2017 www.insightsonindia.com www.insightsias.com TEST – 5 Solutions

Justification: While Harappa and Mohenjodaro are situated in Pakistan, the important sites excavated in India are Lothal and Dholavira in , Rakhigarhi in Haryana, Ropar in the Punjab, Kalibangan and Balathal in Rajasthan.

Learning: Kalibangan has given the evidence of the earliest (2800 BC) ploughed agricultural field ever revealed through an excavation as per Dr. B Lal. It is also a site which has given an evidence of earliest recorded “Earthquake”.

Q Source: Page 9: Introduction to Indian Art – Class XI

53. Consider the following about a city in Madhya Pradesh which held strategic and recreational importance during the Mughal rule. 1. Its natural defence due to Malwa and Narmada encouraged consistent habitations by Parmara Rajputs, Afghans and Mughals. 2. It was the capital city of Ghauri Dynasty founded by Hoshang Shah.

The above refers to?

a) Shahjahanabad b) Dantel c) Bhopal d) Mandu Solution: d)

Learning: The city of Mandu is located near Indore, at an elevation of over 2000 feet and overlooks the Malwa Plateau to the north and the Narmada valley to the south.

• As the capital city of Ghauri Dynasty (1401–1561) founded by Hoshang Shah it acquired a lot of fame. Subsequently, Mandu was

INSIGHTS PRELIMS TEST SERIES 2017 www.insightsonindia.com www.insightsias.com TEST – 5 Solutions

associated with the romance of Sultan Baz Bahadur and Rani Rupmati. • The Mughals resorted to it for pleasure during the monsoon season. • Mandu is a typical representation of the medieval provincial style of art and architecture. • The famous Jahaaz Mahal is located in Mandu. It is an elegant two-storey ‘shippalace’ between two reservoirs.

Q Source: Page 116-117: Introduction to Indian Art – Class XI

54. The Union Cabinet has approved the proposal of Ministry of Finance to merge the Railway budget with the General budget. How this decision will impact Indian Railways (IR)? 1. IR will cease to be a departmentally run commercial undertaking. 2. It will save Railways from the liability of payment of annual dividend to the Central government. 3. The Ministry of Railways will stop receiving Gross Budgetary Support (GBS) from the government.

Select the correct answer using the codes below.

a) 1 and 3 only b) 1 and 2 only c) 2 only d) None of the above Solution: c)

Justification: The merger has been approved with the following administrative and financial arrangements of Railways

INSIGHTS PRELIMS TEST SERIES 2017 www.insightsonindia.com www.insightsias.com TEST – 5 Solutions

• It will continue as a departmentally run commercial undertaking to maintain its distinct entity as at present. So, 1 is wrong.

• It will retain its functional autonomy and delegation of financial powers etc. as per the existing guidelines.

• Its existing financial arrangements will continue wherein it will meet the revenue expenditure from revenue receipts.

• The Capital at charge estimated at more than 2 lakh crore rupees on which Railways pays annual dividend will be wiped off.

• Consequently, Railways will not have dividend liability from 2017- 18 and the Union Ministry of Railways will get Gross Budgetary support.

• It will save Railways from the liability of payment of approximately 9,700 crore rupees as an annual dividend to the Central Government.

Q Source: Union Cabinet Decision to merge the Budgets

55. Lapis lazuli is a rich blue semi-precious stone sometimes flecked with gold. In medieval India, it was used in ornaments and official seals. Where the stone was usually sourced from? a) Java b) Northern Afghanistan c) China d) Western Ghats, India Solution: b)

Learning: The main source in the ancient world was the mountains of Badakshan, northern Afghanistan, from where it was traded widely. Lapis lazuli was used as inlay in ornaments, jewellery, seals, etc

INSIGHTS PRELIMS TEST SERIES 2017 www.insightsonindia.com www.insightsias.com TEST – 5 Solutions

Today mines in northeast Afghanistan and Pakistan are still the major source of lapis lazuli.

Important amounts are also produced from mines west of Lake Baikal in Russia, and in the Andes mountains in Chile. Smaller quantities are mined in Italy, Mongolia, the United States and Canada.

It was used by some of the most important artists of the Renaissance and Baroque, including Masaccio, Perugino, Titian and Vermeer,

Q Source: Glossary: Introduction to Indian Art – Class XI

56. The East Coast Economic Corridor (ECEC) of India runs from a) Kolkata to Tuticorin b) Vishakapatnam to Chennai c) Paradip to Rameshwaram d) Haldia to Paradip Solution: a)

Learning: The ECEC will extend from Kolkata (West Bengal) in the northeast of India to Tuticorin () near the southern-most point of the country.

The Visakhapatnam-Chennai Industrial Corridor (VCIC) section of the East Coast Economic Corridor (ECEC) connects four economic hubs and nine industrial clusters. It will be first industrial corridor developed along India’s coast.

Q Source: ADB has approved $631 million loan for building India’s first coastal industrial corridor between Visakhapatnam (Vizag) and Chennai

INSIGHTS PRELIMS TEST SERIES 2017 www.insightsonindia.com www.insightsias.com TEST – 5 Solutions

57. Consider the following about the Stockholm Convention on Persistent Organic Pollutants (POPs). 1. It is a legally binding instrument on member nations for targeting POPs. 2. The Global Environmental Facility (GEF) is the designated interim financial mechanism for the Stockholm Convention. 3. The term “Dirty Dozen” is often associated with the convention.

Select the correct answer using the codes below.

a) 1 only b) 2 and 3 only c) 1 and 3 only d) 1, 2 and 3 Solution: d)

Justification: Statement 1: It aims to eliminate or restrict the production and use of persistent organic pollutants (POPs) and imposes certain obligation on party states to the convention.

For e.g. developed countries should provide new and additional financial resources and measures to eliminate production and use of intentionally produced POPs, eliminate unintentionally produced POPs where feasible, and manage and dispose of POPs wastes in an environmentally sound manner.

Statement 2: The GEF also serves as the financial mechanism for several other conventions, for e.g. CBD, UNFFFC, UNCCD etc.

Statement 3: The Stockholm Convention established an initial list of 12 key POPs chemicals (the so called dirty dozen) for which signatories are required to reduce the risks to human health and the environment arising from their release.

Q Source: http://www.moef.nic.in/sites/default/files/treaties/international- treaties.html

INSIGHTS PRELIMS TEST SERIES 2017 www.insightsonindia.com www.insightsias.com TEST – 5 Solutions

58. New York Declaration, 2016 was adopted to strengthen the regime of a) Refugee and migrant protection b) Climate Change mitigation c) Global Financial Architecture d) Cross-border trade and investments Solution: a)

Learning: World leaders recently adopted New York Declaration for Refugees and Migrants at the United Nations General Assembly (UNGA).

The declaration expresses the political will of world leaders to protect the rights of refugees and migrants to save lives and share responsibility for large movements on a global scale.

By adopting it UN member states are making bold commitments to

• Develop guidelines on the treatment of migrants in vulnerable situations.

• Start negotiations leading to an international conference and adoption of global compact for safe, regular and orderly migration in 2018.

• Achieve a more equitable sharing of the burden and responsibility for hosting and supporting the world’s refugees by adopting global compact on refugees in 2018.

Q Source: https://www.theguardian.com/commentisfree/2016/sep/19/the- guardian-view-on-the-new-york-declaration-better-than-nothing

INSIGHTS PRELIMS TEST SERIES 2017 www.insightsonindia.com www.insightsias.com TEST – 5 Solutions

59. Which of the following statements about Barak-8 missile is INCORRECT? a) It has been developed jointly by India and Russia. b) It is a long-range nuclear capable ballistic missile. c) It is capable to neutralize aerial targets. d) None of the above Solution: a)

Learning: Barak-8 (Lightning 8 in Hebrew) is long-range nuclear capable ballistic missile, developed jointly by Israel and India.

It has the capacity to identify and neutralize various forms of aerial threats such as rockets, UAVs, planes, helicopters in a single flight. Barak-8 missile’s most technologically advanced aspect is its ability to intercept missiles aimed at sea-bound vessels.

Q Source: http://indianexpress.com/article/india/india-news- india/test-firing-of-barak-8-missile-to-continue-for-second-day- 3042049/

60. “PARAM ISHAN” recently in news is a a) Missile interceptor b) Supercomputer c) Solar powered spacecraft d) Unmanned Aerial Vehicle Solution: b)

Learning: PARAM ISHAN is the fastest and most powerful computer in North East, Eastern and Southern region of India outside Bengaluru (Karnataka).

INSIGHTS PRELIMS TEST SERIES 2017 www.insightsonindia.com www.insightsias.com TEST – 5 Solutions

PARAM ISHAN PARAM ISHAN has been jointly developed by IIT Guwahati and C -DAC (Centre for Development of Advanced Computing).

It can be used research initiatives such as computational chemistry, computational electromagnetic, computational fluid dynamics, civil engineering structures, nana-block self-assemble, optimization etc. It can be also used for Weather, climate modelling and seismic data processing.

Q Source: PIB: Inauguration by MHRD

61. “Special Category of Punishment” mentioned in a recent ruling of the Supreme Court applies to a) Cases where certiorari was issued b) Arbitration cases where any one of the parties declined to accept arbitration award c) All cases handled by fast track courts d) Punishments more severe than life imprisonment but less severe than capital punishment Solution: d)

Learning: This special category of Punishment for the first time was mentioned in the Swami Shraddananda versus State of Karnataka judgment of the Supreme Court in 2008.

• The Supreme Court has moved away from death penalty (capital punishment) after it commuted the death sentence of a youth for raping and killing a seven-year-old girl with the Special Category of Punishment.

• Actual life imprisonment is only 14 years of imprisonment. The SC in its judicial innovation awarded the convict 25 year jail term considering it appropriate punishment between death sentence and life imprisonment in heinous crimes.

INSIGHTS PRELIMS TEST SERIES 2017 www.insightsonindia.com www.insightsias.com TEST – 5 Solutions

Q Source: http://indianexpress.com/article/india/india-news- india/jabalpur-rape-case-sc-commutes-death-penalty-of-convict- 3034714/

62. The objective of the Rotterdam Convention is to promote shared responsibility and cooperative efforts among Parties in the international trade of a) Hazardous chemicals b) Rare species c) Environmental antiques d) Carbon credit Solution: a)

Learning: The Convention covers pesticides and industrial chemicals that have been banned or severely restricted for health or environmental reasons by Parties and which have been notified by Parties for inclusion.

The convention has roots in initiatives of UNEP and FAO.

FAO launched the International Code of Conduct on the Distribution and Use of Pesticides in 1985 and UNEP established the London Guidelines for the Exchange of Information on Chemicals in International Trade in 1987.

On the lines of these initiatives, the convention entered into force in 2004 and became legally binding for its Parties.

Q Source: http://www.pic.int/TheConvention/Overview/tabid/1044/language/en- US/Default.aspx

INSIGHTS PRELIMS TEST SERIES 2017 www.insightsonindia.com www.insightsias.com TEST – 5 Solutions

63. Autotrophic Nutrition is common in a) Plants, fungi and protozoa b) Bacteria, Virus and Protozoa c) Plants and blue-green algae d) Virus and Fungi Solution: c)

Learning: Carbon and energy requirements of the autotrophic organism are fulfilled by photosynthesis.

It is the process by which autotrophs take in substances from the outside and convert them into stored forms of energy.

Cyanobacteria (blue-green algae) and plants both obtain their energy through photosynthesis.

Fungi are saprophytes that live on dead or decaying organic matter.

Q Source: Page 93: Science Xth Standard NCERT

64. The major international protocols that address genetically modified organisms (GMOs) is/are 1. Cartagena protocol 2. Nagoya-Kuala Lumpur Supplementary Protocol 3. Helsinki Convention 4. Geneva Protocol

Select the correct answer using the codes below.

a) 1 and 2 only b) 2 and 3 only c) 1 and 3 only d) 1, 2 and 4 only Solution: a)

INSIGHTS PRELIMS TEST SERIES 2017 www.insightsonindia.com www.insightsias.com TEST – 5 Solutions

Justification: Statement 1: India is a party to the United Nations Convention on Biological Diversity signed at Rio de Janeiro.

The Cartagena Protocol on Biosafety (CPB), the first international regulatory framework for safe transfer, handling and use of Living Modified Organisms (LMOs) was negotiated under the aegis of the Convention on Biological Diversity (CBD).

Statement 2: Following years of negotiations over the question of liability for GMO-produced damages, in 2010, the Nagoya-Kuala Lumpur Supplementary Protocol on Liability and Redress to the Cartagena Protocol on Biosafety (the Supplementary Protocol) was adopted.

India has already ratified the protocol in 2014, you can see here http://pib.nic.in/newsite/PrintRelease.aspx?relid=110927

Statement 3: It is the Convention on the Protection of the Marine Environment of the Baltic Sea Area.

Statement 4: Geneva Protocol addresses “Prohibition of the Use in War of Asphyxiating, Poisonous or other Gases and of Bacteriological Methods of Warfare”.

Q Source: http://www.moef.nic.in/sites/default/files/treaties/international- treaties.html

65. What is common between Latina, Phamsana, Shekhari and Valabhi? a) They are places visited by Buddha during his lifetime. b) All of them host pre-historic caves made during the Pleistocene age. c) They are styles of temple architecture of Northern India. d) None of the above Solution: c)

INSIGHTS PRELIMS TEST SERIES 2017 www.insightsonindia.com www.insightsias.com TEST – 5 Solutions

Learning: The main styles of North Indian temples include Latina, Phamsana, Shekhari and Valabhi types.

• Valabhi style of temples comprises ayatakara talachhanda yojana (rectangular ground plan), a doorway to one of its longer parasvas (side) and a semi- cylindrical sikhara (superstructure).

• About fifty percent of Valabhi style temples are in Uttarakhand.

• Latina types are mainly used for housing the garbhagriha.

• Phamsana type shikhara are broader and shorter than Latina type.

• In many north Indian temples, the phamsana type is used for mandapas while the main garbhagriha is housed in a Latina building.

Q Source: Improvisation: Chapter 6: Introduction to Indian Art – Class XI

66. What do you understand by the Dual nature of light? a) Light is both a wave and a particle. b) Light always travels in a straight line. c) Light travels without being refracted through a transparent medium. d) Light can exhibit wavelengths from both ends of the electromagnetic spectrum. Solution: a)

Learning: Light has a dual nature

• Sometimes it behaves like a particle (called a photon), which explains how light travels in straight lines

• Sometimes it behaves like a wave, which explains how light bends (or diffracts) around an object. So, option B is wrong.

INSIGHTS PRELIMS TEST SERIES 2017 www.insightsonindia.com www.insightsias.com TEST – 5 Solutions

Together Max Planck and Albert Einstein explained the photoelectric effect by assuming that light was actually a stream of little particles, or packets of energy known as photons or quanta.

Scientists now believe that light is both a wave and a particle – a property which they term the wave-particle duality.

Q Source: Page 160: Science Xth Standard NCERT

67. Search-lights and vehicle headlights get powerful parallel beams of light by using a) Convex mirrors b) Polar mirrors c) Biofocal mirrors d) Concave mirrors Solution: d)

Learning: A ray passing through the principal focus of a concave mirror or a ray which is directed towards the principal focus of a convex mirror, after reflection, will emerge parallel to the principal axis.

So, concave mirrors are commonly used in torches, search-lights and vehicles headlights to get powerful parallel beams of light. They are often used as shaving mirrors to see a larger image of the face. The dentists use concave mirrors to see large images of the teeth of patients.

Large concave mirrors are used to concentrate sunlight to produce heat in solar furnaces.

Q Source: Page 165: Science Xth Standard NCERT

INSIGHTS PRELIMS TEST SERIES 2017 www.insightsonindia.com www.insightsias.com TEST – 5 Solutions

68. The Intergovernmental Panel on Climate Change (IPCC) is the leading international body for the assessment of climate change. Consider the following about it. 1. Its membership is open to all member countries of the United Nations (UN). 2. It was established by the United Nations Framework Convention on Climate Change (UNFCCC). 3. Scientists from all over the world contribute to the work of the IPCC on a voluntary basis without receiving pecuniary incentives. 4. It publishes “Assessments Reports” on an annual basis addressing climate change related issues.

Select the correct answer using the codes below.

a) 1 and 3 only b) 2 and 4 only c) 4 only d) 1, 2, 3 and 4 Solution: a)

Justification: Statement 1: As an intergovernmental body, membership of the IPCC is open to all member countries of the United Nations (UN) and WMO. Currently 195 countries are Members of the IPCC. Governments participate in the review process and the plenary Sessions.

Statement 2: It was established by the United Nations Environment Programme (UNEP) and the World Meteorological Organization (WMO) in 1988 to provide the world with a clear scientific view on the current state of knowledge in climate change and its potential environmental and socio-economic impacts.

Statement 3: The Intergovernmental Panel on Climate Change is a huge and yet very small organization. Thousands of scientists from all over the world contribute to the work of the IPCC on a voluntary basis as authors, contributors and reviewers. None of them is paid by the IPCC. The work of the IPCC is guided by a set of principles and procedures.

INSIGHTS PRELIMS TEST SERIES 2017 www.insightsonindia.com www.insightsias.com TEST – 5 Solutions

Statement 4: The first report was published in 1990; second in 1995; third in 2001; fourth in 2007; and Fifth very recently in 2013-14.

These are published materials composed of the full scientific and technical assessment of climate change.

Q Source: Improvisation: http://www.moef.nic.in/sites/default/files/treaties/international- treaties.html + Often in news

69. These bacteria are the most common cause of traveller's diarrhoea as transmission usually occurs through contaminated food and water. Some of its serotypes are harmless and cause gastroenteritis in humans. It is? a) Salmonella Typhi b) Escherichia coli c) Shigella d) Listeria monocytogenes Solution: b)

Learning: E. coli are a large and diverse group of bacteria. Although most strains of E. coli are harmless, others can make you sick. Some kinds of E. coli can cause diarrhea, while others cause urinary tract infections, respiratory illness and pneumonia.

• Most E. coli strains are harmless, but some serotypes can cause serious food poisoning in their hosts, and are occasionally responsible for product recalls due to food contamination.

• The harmless strains are part of the normal flora of the gut, and can benefit their hosts by producing vitamin K2, and preventing colonization of the intestine with pathogenic bacteria.

INSIGHTS PRELIMS TEST SERIES 2017 www.insightsonindia.com www.insightsias.com TEST – 5 Solutions

• E. coli is expelled into the environment within fecal matter. So, high concentration of E. Coli in river waters is an indicator of large dumping of faecal matter.

Q Source: Syllabus: Important microorganisms – Bacteria, Fungi, Virus etc.

70. The Sun is visible to us about 2 minutes before the actual sunrise, and about 2 minutes after the actual sunset because of a) Dispersion of sunlight b) Total Internal Reflection of Sunlight c) Atmospheric refraction d) Constructive interference of Sun light Solution: c)

Learning: By actual sunrise, we mean the actual crossing of the horizon by the Sun.

Figure below shows the actual and apparent positions of the Sun with respect to the horizon.

The time difference between actual sunset and the apparent sunset is about 2 minutes. The apparent flattening of the Sun’s disc at sunrise and sunset is also due to the same phenomenon.

Q Source: Page 195: Science Xth Standard NCERT

INSIGHTS PRELIMS TEST SERIES 2017 www.insightsonindia.com www.insightsias.com TEST – 5 Solutions

71. Nettle leaves have stinging hair, which cause painful stings when touched accidentally. Despite giving pain, stinging nettle is useful as? a) Analgesic b) Antioxidant c) Antimicrobial agent d) All of the above Solution: d)

Learning: Stinging nettle contains a number of chemicals, such as serotonin, histamine and acetylcholine, some of which can be very irritating.

According to the University of Maryland Medical Center, the plant has been used most commonly throughout history as a diuretic and for treating painful muscles and joints, eczema, arthritis, gout, and anemia. Today, it’s used primarily to treat urinary issues, as well as allergies and joint pain.

Studies have shown stinging nettle has antioxidant, antimicrobial, anti- ulcer, astringent and analgesic capabilities

Q Source: Improvisation: Page 28: Science Xth Standard NCERT

72. Bulbs are usually filled with chemically inactive nitrogen and argon gases to 1. Prolong the life of the filament 2. Cut down the current-voltage conversion significantly in the bulb

INSIGHTS PRELIMS TEST SERIES 2017 www.insightsonindia.com www.insightsias.com TEST – 5 Solutions

Which of the above is/are correct?

a) 1 only b) 2 only c) Both 1 and 2 d) None Solution: a)

Justification: The electric heating is also used to produce light, as in an electric bulb. Here, the filament must retain as much of the heat generated as is possible, so that it gets very hot and emits light. It must not melt at such high temperature.

A strong metal with high melting point such as tungsten (melting point 3380°C) is used for making bulb filaments. The filament should be thermally isolated as much as possible, using insulating support, etc.

The bulbs are usually filled with chemically inactive nitrogen and argon gases to prolong the life of filament. Most of the power consumed by the filament appears as heat, but a small part of it is in the form of light radiated. So, 1 is correct. 2 is an absurd statement.

Q Source: Page 218: Science Xth Standard NCERT

73. The Namami Gange Scheme builds over the older Ganga Action Plan. The stated objective(s) of the Ganga Action Plan was to a) Restore the river water quality to the 'Bathing Class' standard b) Restoring the river water quality to the 'Drinking Class' standard c) Bring the major distributaries to ‘Industrial Class’ Standard d) Bring the major tributaries to ‘Near Source’ standard Solution: a)

INSIGHTS PRELIMS TEST SERIES 2017 www.insightsonindia.com www.insightsias.com TEST – 5 Solutions

Learning: The other objectives of the Ganga Action Plan were as under:

• Control of non-point pollution from agricultural run-off, human defecation, cattle wallowing etc

• Research and Development to conserve the biotic diversity of the river to augment its productivity.

• New technology of sewage treatment like Up-flow Anaerobic Sludge Blanket (UASB) and sewage treatment through afforestation

• Rehabilitation of soft-shelled turtles for pollution abatement of river

• Resource recovery options like production of methane for energy generation and use of aquaculture for revenue generation

• To act as trend setter for taking up similar action plans in other grossly polluted stretches in other rivers.

Q Source: Improvisation: Page 267: Science Xth Standard NCERT + Old schemes that are related to current affairs are also asked by UPSC

74. The material used in this school was the spotted red sandstone. This school art reached its peak during the Gupta period in 6th or 7th century. The art school was? a) Gandhara School b) Mathura School c) Pahari School d) Vaishnav School Solution: b)

INSIGHTS PRELIMS TEST SERIES 2017 www.insightsonindia.com www.insightsias.com TEST – 5 Solutions

Learning: Gandhara and Mathura School of Art flourished mainly during reign of Kushana emperor Kanishka during the first century AD. Mathura School had developed indigenously, as one of its distinguishing features.

The Mathura school images include those of Buddha, Bodhisattvas, Vishnu, Shiva, Yakshas, Yakshinis, Jinas etc. representing its vitality and assimilative character as a result of the religious zeal of Brahmanism, Jainism and Buddhism.

Jina Image and Indigenous style of Buddha’s image was a remarkable feature of Mathura art.

Q Source: Q4: SET A: CAPF 2014

75. The Green imperial pigeon is the state bird of a) Mizoram b) Tripura c) Jammu & Kashmir d) Arunachal Pradesh Solution: b)

Learning: It is widely distributed throughout India in the evergreen and moist deciduous biotopes of eastern Uttar Pradesh, Bihar, Orissa and the Eastern Ghats of Andhra Pradesh.

It is also found in the Western Ghats from Kanyakumari to Bombay and in the islands of Andaman and Nicobar. There are a number of subspecies.

This species is found abundantly in their habitats. There is no threat to this species except from habitat loss.

Q Source: Syllabus: State birds, animals, trees and flowers

INSIGHTS PRELIMS TEST SERIES 2017 www.insightsonindia.com www.insightsias.com TEST – 5 Solutions

76. Which of the following gases is the largest component of Biogas? a) Methane b) Hydrogen Sulphide c) Nitric Oxide d) Carbon dioxide Solution: a)

Learning: Bio-gas is an excellent fuel as it contains up to 75% methane. It burns without smoke, leaves no residue like ash in wood, charcoal and coal burning.

Its heating capacity is high and generates gases like methane, carbon dioxide, hydrogen and hydrogen sulphide during heating.

India is rich in biomass and has a potential of 16,881 MW (agro-residues and plantations), 5000 MW (bagasse cogeneration) and 2700 MW (energy recovery from waste).

Q Source: Page 247: Science Xth Standard NCERT

77. Consider the following statements.

Assertion (A): The energy pyramid of an ecosystem is always upright and narrows to the top.

Reason (R): The biomass in the upper trophic levels is generally very high as compared to the lower trophic levels.

In the context of the above, which of these is correct?

a) A is correct, and R is an appropriate explanation of A. b) A is correct, but R is not an appropriate explanation of A.

INSIGHTS PRELIMS TEST SERIES 2017 www.insightsonindia.com www.insightsias.com TEST – 5 Solutions

c) A is correct, but R is incorrect. d) Both A and R are incorrect. Solution: c)

Justification: The green plants in a terrestrial ecosystem capture about 1% of the energy of sunlight that falls on their leaves and convert it into food energy.

• When green plants are eaten by primary consumers, a great deal of energy is lost as heat to the environment, some amount goes into digestion and in doing work and the rest goes towards growth and reproduction. • An average of 10% of the food eaten is turned into its own body and made available for the next level of consumers. • Since each higher trophic level receives only a fraction of energy of the lower trophic levels, the energy pyramid is narrow at the top. So, A is correct. • But, generally (barring some aquatic ecosystems) lower trophic levels have higher biomass as compared to the higher trophic levels. So, R is incorrect. Moreover, even if this were to be so, it doesn’t explain A.

Q Source: Page 261: Science Xth Standard NCERT

78. Consider the following statements.

Assertion (A): Continuous passage of electric current in a circuit produces heat.

Reason (R): Electrons are consumed in an electric circuit.

In the context of the above, which of these is correct?

INSIGHTS PRELIMS TEST SERIES 2017 www.insightsonindia.com www.insightsias.com TEST – 5 Solutions

a) A is correct, and R is an appropriate explanation of A. b) A is correct, but R is not an appropriate explanation of A. c) A is correct, but R is incorrect. d) A is incorrect, but R is correct. Solution: c)

Justification: Many people think that electrons are consumed in an electric circuit. This is wrong.

We pay the electricity board or electric company to provide energy to move electrons through the electric gadgets like electric bulb, fan and engines.

We pay for the energy that we use, not the number of electrons we consume. So, R is clearly wrong.

Q Source: Page 219: Science Xth Standard NCERT

79. Which of the following differentiate between Buddhist rock- cut caves Viharas and Chaityas? 1. Viharas generally contain relics of Buddha, Chaityas do not. 2. Viharas were constructed for shelter; Chaityas were used as prayer halls.

Which of the above is/are correct?

a) 1 only b) 2 only c) Both 1 and 2 d) None Solution: b)

Justification: Statement 1: Buddhist relics are generally housed in Stupas. Viharas do not contain stupas, Chaityas do.

INSIGHTS PRELIMS TEST SERIES 2017 www.insightsonindia.com www.insightsias.com TEST – 5 Solutions

Statement 2: Viharas: To begin with, both Jaina and Buddhist monks went from place to place throughout the year, teaching people.

The only time they stayed in one place was during the rainy season, when it was very difficult to travel. Then, their supporters built temporary shelters for them in gardens, or they lived in natural caves in hilly areas.

As time went on, many supporters of the monks and nuns, and they themselves, felt the need for more permanent shelters and so monasteries were built. These were known as viharas. They do not contain stupas.

Chaityas: Chaitya grihas or halls of worship were built all over the country either of brick or excavated from rocks.

Both early Chaityas and Viharas were made by woods and later stone-cut Chaityas and Viharas were made.

Chaitya was a rectangular prayer hall with a stupa placed in the centre, the purpose was prayer.

Q Source: Page 36: Introduction to Indian Art – Class XI

80. If you stroll inside the Ajanta caves, you are likely to find which of the following famous images/sculpture there? 1. Mahaparinirvana of Buddha 2. Shiva slaying Andhaka and Wedding of Shiva 3. Padmapani and Vajrapani 4. Trimurti, Gangadhara and Ardhanarishvara

Select the correct answer using the codes below.

a) 2 and 3 only b) 1, 2 and 4 only c) 1 and 3 only

INSIGHTS PRELIMS TEST SERIES 2017 www.insightsonindia.com www.insightsias.com TEST – 5 Solutions

d) 1, 2, 3 and 4 Solution: c)

Justification: Statement 1: Mahaparinirvana of Buddha on the right aisle wall and the assault of Mara during Buddha’s penance adorns the same wall. The Mahaparinirvana of the Buddha is when he finally achieves release from the mortal world.

Statement 2 and 4: These are found in Elephanta caves. Described as a "masterpiece of Gupta-Chalukyan art", the most important sculpture in the Elephanta caves is the Trimurti. The carved panel facing this one is a two-level depiction of Ravana lifting Kailash.

Statement 3: Seated Buddha in Dharmachakrapravartana mudra is notable sculpture while the notable paintings include Padmapani and Vajrapani.

Q Source: Page 36: Introduction to Indian Art – Class XI

81. Project 15B is related to the development of a) Modern war ships b) Drought resistant seeds c) Solar powered spacecrafts d) Public telecommunications spectrum Solution: a)

Learning: Union Government in 2011 had sanctioned four 15B ships at to develop a class of stealth guided missile destroyers for the Indian Navy.

INSIGHTS PRELIMS TEST SERIES 2017 www.insightsonindia.com www.insightsias.com TEST – 5 Solutions

• The Project 15B missile destroyers are modern warships equipped with latest weapons package in continuation of lineage of the highly successful Delhi and Kolkata Class ships.

• The first ship of the project, guided-missile destroyer ‘Visakhapatnam’ was launched in 2015.

• Recently, Indian Navy’s indigenously built most Advanced Guided Missile Stealth Destroyer ‘Mormugao’ was launched. It belongs to Visakhapatnam class of ships being constructed under Project 15B.

Q Source: http://timesofindia.indiatimes.com/india/Mormugao- -second-guided-missile-destroyer-launched-in- Mumbai/articleshow/54376958.cms

82. Which of the following is/are the key features of Federalism? 1. Different tiers of government govern the same citizens, but each tier has its own jurisdiction. 2. Each tier of government must draw all its financial resources independent of the other tier. 3. The existence and authority of each tier of government generally is constitutionally guaranteed. 4. Division of powers between State units cannot be arbitrarily manipulated by any one tier of government alone.

Select the correct answer using the codes below.

a) 1, 2 and 3 only b) 1, 3 and 4 only c) 3 and 4 only d) 1 and 2 only Solution: b)

Justification: Statement 1: Each tier has its own jurisdiction in specific matters of legislation, taxation and administration; for e.g. in

INSIGHTS PRELIMS TEST SERIES 2017 www.insightsonindia.com www.insightsias.com TEST – 5 Solutions

India states legislate in matters of police, and Centre legislates in areas of national security as a whole.

Statement 2: Tiers may be dependent upon each other; however not completely, else it erodes autonomy. So, 2 is incorrect.

Statement 4: The fundamental provisions of the constitution cannot be unilaterally changed by one level of government. Such changes require the consent of both the levels of government.

Q Source: Page 15: Democratic Politics: Class Xth NCERT

83. Aqua regia is one of the few reagents that is able to dissolve gold and platinum. It is a mixture of a) Concentrated hydrochloric acid and concentrated nitric acid b) Milk of Magnesia and Concentrated Sodium Hydroxide c) Magnesium Hydroxide, Sodium Bicarbonate and Potassium Chloride d) Concentrated Chloroacetic acid and Carboxylic acid Solution: a)

Learning: It is a very strong acid. It is made by mixing one part nitric acid and three parts hydrochloric acid.

• The acid was named by alchemists because it can dissolve the noble metals gold and platinum. Tantalum, iridium and a few other metals are not dissolved by it.

• It is also used to clean certain lab machines from tiny metal particles. It is particularly used in the purification and extraction of gold and platinum.

• It has an interesting history to it.

• When Germany invaded Denmark in World War II, the Hungarian chemist George de Hevesy took the Nobel Prize medals of Max von

INSIGHTS PRELIMS TEST SERIES 2017 www.insightsonindia.com www.insightsias.com TEST – 5 Solutions

Laue and James Franck and dissolved them in Aqua regia so that Germans cannot take these medals.

• Later he submitted the dissolved Gold to the Royal Swedish Academy which again made medals for the Nobel laureates from the gold.

Q Source: Page 44: Science Xth Standard NCERT

84. If State laws on subjects mentioned in the Concurrent List conflict with a Central Law, which of these follows? a) The state law prevails over the Central law if the State legislature passes it again with special majority. b) The Central law prevails over the State law. c) The matter is moved to the Supreme Court which decides on the validity of the respective laws in the larger national interest. d) The matter is referred to the Governor whose decision shall be final and binding in this regard. Solution: b)

Justification: The Constitution provides a scheme for demarcation of powers through three ‘lists’ in the seventh schedule.

But, the Constitution also provides primacy to Parliament on concurrent list items: if there is a conflict, a central law will override a state law, i.e. the provisions of the state law will not take effect.

Q Source: Page 17: Democratic Politics: Class Xth NCERT

85. The Lieutenant Governor of Delhi is appointed by a) The President on the advice of the Ministry of Home Affairs

INSIGHTS PRELIMS TEST SERIES 2017 www.insightsonindia.com www.insightsias.com TEST – 5 Solutions

b) The Legislative Assembly of Delhi on the advice of the UT Council of Ministers c) The Chief Minister of Delhi after consulting a collegium consisting of the Chief Justice of the Delhi High court and other senior judges d) The Delhi Development Authority (DDA) after obtaining consent from the President of India Solution: a)

Learning: Lt. Governor is the ‘administrator’ of Delhi appointed by the President at the recommendation of Union Home Ministry.

Specifically for New Delhi, Lt Governor has additional powers (land, law and order etc) considering the strategic importance of capital New Delhi.

All acts passed by the legislative assembly require Lt Governor’s approval.

Where there is a difference of opinion between assembly and the Lt Governor, the matter is referred to the president.

Q Source: Current Affairs: Improvisation: Page 17: Democratic Politics: Class Xth NCERT

86. Consider the following about the Merchandise Exports from India Scheme (MEIS). 1. It was launched under Foreign Trade Policy of India (FTP) 2015-20. 2. It is a sub-scheme of “Exports from India Scheme”. 3. It aims to offset infrastructural inefficiencies and associated costs involved in export of goods and products.

Select the correct answer using the codes below.

a) 1 only b) 2 and 3 only

INSIGHTS PRELIMS TEST SERIES 2017 www.insightsonindia.com www.insightsias.com TEST – 5 Solutions

c) 3 only d) 1, 2 and 3 Solution: d)

Justification:

MEIS has replaced following 5 different schemes of earlier FTP:-

• Focus Product Scheme (FPS)

• Market-Linked Focus Product Scheme (MLFPS)

• Agri-Infrastructure Incentive Scrip

• Vishesh Krishi Gram Udyog Yojana (VKGUY) It is one of the two schemes introduced in FP 2015-20, as a part of Exports from India Scheme. The other scheme is Service Exports from India Scheme (SEIS).

Objective of MEIS is to offset infrastructural inefficiencies and associated costs involved in export of goods and products, which are produced and manufactured in India.

It seeks to enhance India’s export competitiveness of these goods and products having high export intensity, employment potential.

Q Source: http://www.business-standard.com/article/news- cm/government-announces-enhanced-support-under-merchandise- exports-from-india-scheme-meis-of-the-foreign-trade-policy- 116092200688_1.html

87. Consider the following statements about local self- government in India. 1. It is constitutionally mandatory to hold regular elections to local government bodies.

INSIGHTS PRELIMS TEST SERIES 2017 www.insightsonindia.com www.insightsias.com TEST – 5 Solutions

2. An independent institution, State Election Commission, has been created in each State to conduct panchayat and municipal elections.

Which of the above is/are correct?

a) 1 only b) 2 only c) Both 1 and 2 d) None Solution: c)

Learning: Seats are reserved in the elected bodies and the executive heads of these institutions for the Scheduled Castes Scheduled Tribes and Other Backward Classes (OBC provision can be made by State legislatures; it is not compulsory). At least one-third of all positions are reserved for women.

The State governments are required to share some powers and revenue with local government bodies. The nature of sharing varies from State to State.

Q Source: Page 24: Democratic Politics: Class Xth NCERT

88. The Prime Minister’s High Level Committee, popularly known as Sachar Committee was setup with a focus on a) Curbing violence and discrimination against Dalits b) Blockades in social and economic development of Women in India and means to empower them c) Social, economic, and educational conditions of the Muslim community of India d) Creating social harmony between the people of North-eastern India and the rest of India

INSIGHTS PRELIMS TEST SERIES 2017 www.insightsonindia.com www.insightsias.com TEST – 5 Solutions

Solution: c)

Learning: The committee was headed by former Chief Justice of the Delhi High Court Rajinder Sachar and had six other members.

• The report was the first of its kind to reveal the economic and social lag of the Muslim community compared with other communities.

• An issue highlighted was that while Muslims constitute 14% of the Indian population, they only comprise 2.5% of the Indian bureaucracy.

• The Sachar Committee concluded that the conditions facing Indian Muslims was below that of Scheduled Castes and Scheduled Tribes.

• In November 2013, the Gujarat government contended before the Supreme Court that the Rajinder Sachar Committee was "unconstitutional," and that it only sought to help Muslims.

• It has strongly criticized the manner in which the PMO set up the Sachar Committee in 2005 to survey the socio-economic conditions of Muslims, while "ignoring" other religious minorities.

Q Source: Page 50: Democratic Politics: Class Xth NCERT

89. The Dhammacakkappavattana Sutta deals with a) Order of the Monasteries, Chaityas and Viharas b) Chantings and prayers compiled by major Buddhist scholars in Tibet and India c) First teaching given by Gautama Buddha after he attained enlightenment d) Interpretations of core Buddhist teachings given in Abhidhamma Pitakas Solution: c)

INSIGHTS PRELIMS TEST SERIES 2017 www.insightsonindia.com www.insightsias.com TEST – 5 Solutions

Learning: The Setting in Motion of the Wheel of the Dharma Sutra or Promulgation of the Law Sutra is considered to be a record of the first teaching given by Gautama Buddha after he attained enlightenment.

• According to tradition, the Buddha gave this teaching in Sarnath, India, to the "five ascetics", his former companions with whom he had spent six years practicing austerities.

• The main topic of this sutra is the Four Noble Truths, which are the central teachings of Buddhism that provide a unifying theme, or conceptual framework, for all of Buddhist thought.

• This sutra also introduces the Buddhist concepts of the Middle Way, impermanence, and dependent origination.

Q Source: Page 20: Introduction to Indian Art – Class XI

90. Which of the following are legally mandatory for political parties in or political candidates fighting elections in India? 1. Giving a certain percentage of election tickets to women candidates 2. A record of the adherence of a political party to its own Constitution 3. Submit an affidavit giving details of his property and criminal cases pending against him to the concerned authority

Select the correct answer using the codes below.

a) 1 and 2 only b) 2 only c) 3 only d) 1 and 3 only Solution: c)

Justification: These are some of the compulsory provisions:

INSIGHTS PRELIMS TEST SERIES 2017 www.insightsonindia.com www.insightsias.com TEST – 5 Solutions

• The Constitution was amended to prevent elected MLAs and MPs from changing parties.

• The Supreme Court passed an order to reduce the influence of money and criminals. Now, it is mandatory for every candidate who contests elections to file an affidavit giving details of his property and criminal cases pending against him.

• The Election Commission passed an order making it necessary for political parties to hold their organisational elections and file their income tax returns

Besides these, many suggestions are often made to reform political parties:

• A law should be made to regulate the internal affairs of political parties. It should be made compulsory for political parties to maintain a register of its members, to follow its own constitution, to have an independent authority, to act as a judge in case of party disputes, to hold open elections to the highest posts.

• It should be made mandatory for political parties to give a minimum number of tickets, about one-third, to women candidates. Similarly, there should be a quota for women in the decision making bodies of the party.

Q Source: Page 85-86: Democratic Politics: Class Xth NCERT

91. Plant tissue culture relies on “totipotency” of plant cells which means the a) Genetic similarity across many breeds of the same plant b) Ability to regenerate a whole plant from plant cells c) Absence of cell walls to facilitate gene transfer mechanism

INSIGHTS PRELIMS TEST SERIES 2017 www.insightsonindia.com www.insightsias.com TEST – 5 Solutions

d) Native reproductive abilities of plants via pollination mechanism Solution: b)

Learning: Plant tissue culture is a collection of techniques used to maintain or grow plant cells, tissues or organs under sterile conditions on a nutrient culture medium of known composition. Plant tissue culture is widely used to produce clones of a plant in a method known as micropropagation.

• Single cells, plant cells without cell walls (protoplasts), pieces of leaves, stems or roots can often be used to generate a new plant on culture media given the required nutrients and plant hormones.

Different techniques in plant tissue culture may offer certain advantages over traditional methods of propagation, including:

• The production of exact copies of plants that produce particularly good flowers, fruits, or have other desirable traits.

• To quickly produce mature plants.

• The production of multiples of plants in the absence of seeds or necessary pollinators to produce seeds.

• The regeneration of whole plants from plant cells that have been genetically modified etc.

Q Source: Page 132: Science Xth Standard NCERT

92. With reference to Jainism, consider the following about Tirthanakars. 1. They delay their enlightenment to liberate earthly beings. 2. As per Jain legend, the birth of Tirthanakaras does not occur in human form. 3. Mahavir was the first Tirthankar.

INSIGHTS PRELIMS TEST SERIES 2017 www.insightsonindia.com www.insightsias.com TEST – 5 Solutions

Select the correct answer using the codes below.

a) 1 and 2 only b) 1 and 3 only c) 2 only d) None of the above Solution: d)

Justification: Statement 1: After achieving enlightenment, a Tirthankara shows others the path to enlightenment. It is said that the inner knowledge of all Tirthankaras is perfect and identical in every respect, for the teachings of one Tirthankara do not contradict those of another.

Statement 2: Tirthankaras are just ordinary man, born as human, but through their intense practice of kindness, equanimity and meditation, they attain the state of a Tirthankara.

A Tirthankar is an individual who destroys attachment with all the earthly things and relations, he frees himself absolutely from ignorance.

He becomes a Sidha and frees himself from the cycle of birth and re- birth.

Statement 3: He was the last tirthanakar. Rishabha was the first.

Q Source: Improvisation: Page 103: Introduction to Indian Art – Class XI

93. Among the Pallava Period bronzes of the eighth century Shiva has been shown in the ardhaparyanka asana and achamana mudra. What do the asana and mudra suggest respectively? 1. He is about to destroy the Universe. 2. He is lifting mount Kailasha. 3. He is opening his third eye. 4. He is about to drink poison.

INSIGHTS PRELIMS TEST SERIES 2017 www.insightsonindia.com www.insightsias.com TEST – 5 Solutions

Select the correct answer using the codes below.

a) 1 and 3 b) 2 and 4 c) 2 and 3 d) None of the above options is correct. Solution: d)

Justification: The icon of Shiva shown in the Pallava period is seated in ardhaparyanka asana (one leg kept dangling). The right hand is in the achamana mudra gesture, suggesting that he is about to drink poison. Since the above mentioned statements contain only one correct statement, D would be the correct answer.

Learning: In other sculptures shown in ancient India, the union of Shiva and Parvati is very ingeniously represented in the ardhanarisvara murti in a single image.

Beautiful independent figurines of Parvati have also been modelled, standing in graceful tribhanga posture.

Q Source: Page 107: Introduction to Indian Art – Class XI

94. In the human body significant magnetic field is produced by which of these organs? a) Spleen and liver b) Heart and Brain c) Stomach and Duodenum d) Hands and legs Solution: b)

Learning: An electric current always produces a magnetic field. Even weak ion currents that travel along the nerve cells in our body produce magnetic fields.

INSIGHTS PRELIMS TEST SERIES 2017 www.insightsonindia.com www.insightsias.com TEST – 5 Solutions

• When we touch something, our nerves carry an electric impulse to the muscles we need to use. This impulse produces a temporary magnetic field.

• These fields are very weak and are about one-billionth of the earth’s magnetic field. Two main organs in the human body where the magnetic field produced is significant, are the heart and the brain. The magnetic field inside the body forms the basis of obtaining the images of different body parts.

• This is done using a technique called Magnetic Resonance Imaging (MRI). Analysis of these images helps in medical diagnosis.

Q Source: Page 232: Science Xth Standard NCERT

95. With reference to the history of Ancient India, who were the Piyadassis? a) Traders who provided services to the mid-town b) Spies of the military department c) King was also called as Piyadassis d) Chroniclers who wrote genealogies Solution: c)

Learning: James Prinsep, an officer in the mint of the East India Company, deciphered Brahmi and Kharosthi, two scripts used in the earliest inscriptions and coins.

• He found that most of these mentioned a king referred to as Piyadassi – meaning “pleasant to behold”; there were a few inscriptions which also referred to the king as Asoka, one of the most famous rulers known from Buddhist texts.

• This gave a new direction to investigations into early Indian political history as European and Indian scholars used inscriptions

INSIGHTS PRELIMS TEST SERIES 2017 www.insightsonindia.com www.insightsias.com TEST – 5 Solutions

and texts composed in a variety of languages to reconstruct the lineages of major dynasties.

• Devanampiya, often translated as “beloved of the gods” was another term attributed to kings.

Q Source: Q3: SET A: CAPF 2014

96. Consider the following about the famous “Objectives Resolution” proposed by Jawahar Lal Nehru. 1. It was proposed to the Constituent Assembly of India before the enactment of the Constitution. 2. It contained clear provisions to abolish Privy Purse from India. 3. It became the basis for adding the words “Socialist” and “Secular” to the Constitution in 1950.

Select the correct answer using the codes below.

a) 1 only b) 2 and 3 only c) 3 only d) None of the above Solution: a)

Justification: Statement 1: It was proposed in 1946, and later passed by the Constituent assembly shaping the Preamble of the constitution.

Statement 2: There was no such provision.

Statement 3: It was added later by the 42nd amendment to the Indian constitution.

This resolution enshrined the aspirations and values behind the Constitution making. On the basis of the Objectives Resolution, India’s Constitution gave institutional expression to the fundamental

INSIGHTS PRELIMS TEST SERIES 2017 www.insightsonindia.com www.insightsias.com TEST – 5 Solutions commitments: equality, liberty, democracy, sovereignty and a cosmopolitan identity.

Q Source: Q15: SET A: CAPF 2014

97. Consider the following statements. 1. North Korea unilaterally withdrew from the Treaty on the Non- Proliferation of Nuclear Weapons (N PT). 2. North Korea recently violated the Comprehensive Nuclear-Test- Ban Treaty (CTBT) by testing nuclear weapons despite being a signatory to the convention.

Which of the above is/are correct?

a) 1 only b) 2 only c) Both 1 and 2 d) None Solution: a)

Justification: Statement 1: It withdrew from NPT in 2003 considering the nuclear expansionism and arm twisting of the United States. It wanted to build a credible deterrence against major powers especially US.

Statement 2: It is not a party to the Comprehensive Nuclear-Test-Ban Treaty (CTBT) or a member of the Missile Technology Control Regime (MTCR).

Learning:

• Also, the DPRK is not a party to the Chemical Weapons Convention (CWC), and is believed to possess a large chemical weapons program.

INSIGHTS PRELIMS TEST SERIES 2017 www.insightsonindia.com www.insightsias.com TEST – 5 Solutions

• North Korea is a party to the Biological and Toxin Weapons Convention (BTWC) and Geneva Protocol, but is suspected of maintaining an offensive weapons program in defiance of the BTWC.

• North Korea's interest in a nuclear weapons program dates to the end of World War II.

• North Korea conducted five nuclear weapons tests in 2006, 2009, 2013, and twice in 2016 claiming that the January 2016 test was a thermonuclear device.

Q Source: North Korea recent nuclear Tests

98. The Taj-i-Izzat invented by Humayun was a) A burqa for subject women b) A royal headdress c) A set of religious instructions targeting the moral decline in the empire d) A code for conduct for female courtesans Solution: b)

Learning: In Humayun’s period, in terms of the accounts of his reign, there are references to garments like qaba, pirahan, jilucha, jiba, kasaba etc.

• These references suggest a strong link with their homeland. Humayun is said to have invented the Taji-izzat, a headdress composed of a cap (kulah) and a wrapping– cloth (asabah). • The cap had an opening in front, thus forming a figure ‘V’. As it had two divisions, each of these when folded upwards, produced the same figure. Thus 77 was formed (in the Arab Alphabet 77 is written as VV), which was equal to the word “zz” in numerical value.

INSIGHTS PRELIMS TEST SERIES 2017 www.insightsonindia.com www.insightsias.com TEST – 5 Solutions

• This may mean the crown of honour, but also the crown of 77, for the numerical value of the letter i is 70 and of the letter Z is seven. • Humayun named it Taj-i-izzat. Q Source: Q28: SET A: CAPF 2014

99. In the “Tribhanga” posture depicted in many famous ancient sculptures a) The dancer turns his head towards her peers in alternate clockwise and anti-clockwise fashion b) Head is inclined to one side while the upper and lower body take opposite directions to each other c) Dancer performs on a metal plate which touches one-third of his feet at any given time d) Body stands gracefully with weight placed on one leg in a meditative posture Solution: b)

Learning: The ideal postures of the body in movement are based upon these Bhangas or bend which represent the change of the body from the central straight line or balance of the figure.

Bhangas are of 4 kinds:

• Abhanga (slight flexion)

• Samabhanga (equipose)

• Atibhanga (excessive flexion)

• Tribhanga (three flexions) Tribhanga posture is the thrice-bent figure in which the head is inclined to one side, the upper body is bent in the opposite direction and part of the body below the waist takes again the reverse direction. The Tribhanga actions are dramatic energetic muscle actions.

INSIGHTS PRELIMS TEST SERIES 2017 www.insightsonindia.com www.insightsias.com TEST – 5 Solutions

The ‘Dancing Girl’ in tribhanga posture from Mohenjodaro is the earliest bronze sculpture datable to 2500 BCE.

Q Source: Improvisation: Page 103: Introduction to Indian Art – Class XI

100. Consider the following statements about the legislature of Delhi. 1. The Lieutenant Governor shall from time-to-time summon the Legislative Assembly to meet at such time and place as he thinks fit. 2. Only the President is authorized to prorogue or dissolve the Delhi Assembly.

Which of the above is/are correct?

a) 1 only b) 2 only c) Both 1 and 2 d) None Solution: a)

Justification: Statement 1: A Bill or amendment shall not be introduced into, or moved in the Legislative Assembly except on the recommendation of the Lieutenant, Governor, if such Bill or amendment makes provision for any of the following matters,

Statement 2: When a Bill has been passed by the Legislative Assembly, it shall be presented to the Lieutenant Governor and the Lieutenant Governor shall declare either that he assents to the Bill or that he withholds assent therefrom or that he reserves the Bill for the consideration of the President.

INSIGHTS PRELIMS TEST SERIES 2017 www.insightsonindia.com www.insightsias.com TEST – 5 Solutions

The Lieutenant Governor may, from time to time prorogue the Assembly or dissolve the Assembly.

Q Source: Current Affairs: Improvisation: Page 17: Democratic Politics: Class Xth NCERT

INSIGHTS PRELIMS TEST SERIES 2017